Пробные егэ каждую неделю по химии 2018: Тренировочный вариант ЕГЭ 2021 №13 по истории с ответами и решениями от 01.03.2021

Содержание

Тренировочный вариант ЕГЭ 2021 №13 по истории с ответами и решениями от 01.03.2021

]]>
11.05.2021

Вариант №13 по истории из серии пробных ЕГЭ от проекта ЕГЭ100баллов https://vk.com/ege100ballov «Пробный ЕГЭ каждую неделю».

Вариант составлен по образцу и примеру демоверсии ФИПИ для ЕГЭ в 2021 году. За основу взяты кодификатор ФИПИ и сама последовательность и типажи заданий по истории.

Ответы представлены в конце документа. Также там есть подробный разбор всех заданий по истории из второй части теста + критерии для проверки.

]]>

Вариант проводился 1 марта 2021 года, в добровольном порядке.

Есть вопросы? Задавайте в комментариях ниже!

Примеры заданий

ЗАДАНИЕ 4

Запишите пропущенное слово.

______________ как вид феодальной земельной собственности с конца XV в. противостояла поместью, а в начале XVIII в. разница между ними была фактически стёрта.

ЗАДАНИЕ 16

Какие суждения, относящиеся к схеме, являются верными? Выберите три суждения из шести предложенных. Запишите в таблицу цифры, под которыми они указаны.

  1. В XII—XV вв. на землях, центрами которых были города «1» и «3», установилась республиканская форма правления.
  2. Современниками обозначенных на схеме событий были хан Батый и князь Даниил Галицкий.
  3. Поход, обозначенный стрелками, подписанными в легенде схемы римской цифрой «1», возглавлял ярл Биргер.
  4. Битва, обозначенная на схеме буквой «А», произошла в летнее время года.
  5. Внуком князя, возглавлявшего походы, обозначенные стрелками, подписанным в легенде схемы римской цифрой «III», был Иван Калита.
  6. Князь, возглавлявший походы, обозначенные стрелками, подписанным в легенде карты римской цифрой «III», прославился военными победами над ордынцами.

Смотреть в PDF:

Или прямо сейчас: cкачать в pdf файле.

Сохранить ссылку:
Добавить комментарий

Комментарии без регистрации. Несодержательные сообщения удаляются.

Тренировочный вариант ЕГЭ 2021 №13 по биологии с ответами и решениями от 22.02.2021

]]>
11.05.2021

Вариант №13 по биологии из серии пробных ЕГЭ от проекта ЕГЭ100баллов https://vk.com/ege100ballov «Пробный ЕГЭ каждую неделю».

Вариант составлен по образцу и примеру демоверсии ФИПИ для ЕГЭ в 2021 году. За основу взяты кодификатор ФИПИ и сама последовательность и типажи заданий по биологии.

Ответы представлены в конце документа. Также там есть подробный разбор всех заданий по биологии из второй части теста + критерии для проверки.

]]>

Вариант проводился 22 февраля 2021 года, в добровольном порядке.

Есть вопросы? Задавайте в комментариях ниже!

Примеры заданий

ЗАДАНИЕ 4

Все приведённые ниже термины, кроме двух, относятся к описанию мейоза. Определите два признака, «выпадающих» из общего списка, и запишите в ответ цифры, под которыми они указаны.

  1. редукционное деление
  2. оплодотворение
  3. онтогенез
  4. гаметогенез
  5. конъюгация

ЗАДАНИЕ 17

Выберите 3 верных ответа из шести, укажите цифры, под которыми они указаны. В каких процессах биосферы происходят окислительно-восстановительные реакции?

  1. хемосинтез
  2. выветривание
  3. метаболизм
  4. вулканизация
  5. опыление
  6. фотосинтез

ЗАДАНИЕ 22

Долгое время считалось, что вислород при фотосинтезе образуется из углекислого газа. Какой метод использовали ученые, чтобы доказать образование кислорода при фотолизе воды? На чем основан это метод?

Смотреть в PDF:

Или прямо сейчас: cкачать в pdf файле.

Сохранить ссылку:
Добавить комментарий

Комментарии без регистрации. Несодержательные сообщения удаляются.

Пробный ЕГЭ математика бесплатно

 

Запишись на наш пробный ЕГЭ онлайн! Опереди других абитуриентов!
Пусть они трясутся от ужаса в ожидании ЕГЭ. Узнай свой реальный уровень прямо сейчас.

Пробные ЕГЭ проводят не во всех школах. Где-то они платные, где-то их нет совсем. Проверяй свои знания как хочешь.
У нас — качественно и бесплатно!

Мы получаем множество писем от наших подписчиков из разных городов России. И все спрашивают нас о том, когда же будет следующий Пробный ЕГЭ Онлайн по математике. Рассказывают, что все доступные варианты уже изучили наизусть, и хочется порешать чего-то свеженького.

Скорее регистрируйтесь!
Задания варианта доступны тем, кто зарегистрировался

Почему вам стоит пройти наш Пробный ЕГЭ Онлайн?

— Задания первой части мы подбираем их так, чтобы каждая задача была ключом к определенной важной теме ЕГЭ по математике. Задания составлены в соответствии с критериями ФИПИ, и при этом решений их в интернете нет.

— Задания второй части – авторские задачи Анны Малковой. Вы не найдете их решений в Интернете! Гугл не поможет! Как на настоящем экзамене.

Дорогие участники! Специально для вас придумала задачи второй части, максимально близкие к официальным пробным 2018 года. Гугл вам не подскажет, потому что этих задач в интернете не найдете. У вас будет вариант Пробного ЕГЭ, который еще никто не видел.  А 4 марта — видеоразбор и награждение победителей!

Анна Малкова.

Первые 50 присланных работ мы проверяем индивидуально. В проверке участвуют лучшие репетиторы Москвы, работающие у нас в «ЕГЭ-Студии».
Разбор задач – 4 марта, на Онлайн Мастер-классе Анны Малковой.
И все это – бесплатно.
Скорее регистрируйся!

Победитель получает ПОДАРКИ от ЕГЭ-Студии.

Занявшие 2 и 3 место тоже получают ПОДАРКИ от ЕГЭ-Студии.

— Видеокурсы «Получи пятерку» и Премиум

— Участие в Годовом Онлайн-курсе и математических тренингах.
— После Пробных ЕГЭ мы традиционно поздравляем чемпионов на нашем сайте. Покажи своей школьной учительнице скриншот сайта, где тебя поздравляют с победой!

Стань Победителем!

Видеоразбор задач — 4 марта онлайн.
А пока – регистрируемся
— и готовимся!
Повторяем формулы
Читаем, как решать задачи ЕГЭ по математике
Изучаем видеокурсы Анны Малковой

ЕГЭ-Студия и Анна Малкова проводят Пробный ЕГЭ онлайн в 12-й раз. Более 150000 участников уже прошли наш пробный ЕГЭ онлайн. Теперь они — студенты ведущих вузов. Не упусти свой шанс!

Акция для учителей математики и директоров школ

Участвовать в нашем Пробном ЕГЭ онлайн теперь можно целыми классами!

Мы выбираем класс, который пришлет больше всего работ. Эти работы я проверю лично. К каждой работе я напишу краткие комментарии.

Класс-победитель получит от ЕГЭ-Студии:

  • Двухчасовое занятие со мной по разбору ошибок и самых интересных задач – онлайн или в нашем офисе в Москве (в обговоренное время).
  •  Один комплект «Получи пятерку» на дисках или электронную версию на выбор
  • Один комплект «Премиум» на дисках или электронную версию на выбор
  • Два доступа к Годовому Онлайн курсу по математике.
  • Сертификаты и поздравления победителям на сайте ЕГЭ-Студии

 

Если ваш класс хочет участвовать в АКЦИИ:

 

  • Каждый участник (ученик) от класса подписывается на рассылку на /free
  • Присылает свою работу с зарегистрированного емейла. В теме письма: ФИО учителя, город, № или название школы, класс, фамилия и имя.
  • Работа присылается (учеником) в виде читабельных фотографий размером не более 500 Кб каждая на адрес [email protected]
  • Учитель присылает на адрес  [email protected] ФИО участников и свои, город, № или название школы, класс.
  • Учитель вправе выставить на конкурс несколько классов и предоставить свою работу для оценки.
  • Список участников и их рейтинги будут опубликованы на специальной странице сайта /
  • Внимание. Одинаковые работы не оцениваются.  Даже если вы пишете Пробный ЕГЭ в школе, всем классом, каждый работает самостоятельно. Как на экзамене!

 

Более 150000 участников уже прошли наш пробный ЕГЭ онлайн. Теперь они — студенты ведущих вузов. Не упусти свой шанс! 

 

Регистрируемся сейчас!

Егэ химия пробные варианты каждую неделю. Подготовка к ЕГЭ по химии – быстро и качественно

Вариант-1

Часть 2.

Для выполнения заданий 30, 31

Оксид кремния ( IV ), фтороводород, гидроксид калия, углерод, фосфат кальция.

30.

31.

32.

33.

34.

35.

Репетиционный тест ЕГЭ по химии в 2018 году

Вариант-2


Часть 2.

Для записи ответов на задания 30-34 используйте БЛАНК ОТВЕТОВ №2. Запишите сначала номер задания, а затем его подробное решение. Ответы записывайте четко и разборчиво.

Для выполнения заданий 30, 31 используйте следующий перечень веществ:

Фосфид кальция, перманганат натрия, оксид азота ( IV ), гидроксид меди ( II ), хлороводород.

Допустимо использование водных растворов веществ.

30. Из предложенного перечня веществ выберите вещества, между которыми может протекать окислительно-восстановительная реакция. В ответе запишите уравнение только одной из возможных окислительно-восстановительных реакций. Составьте электронный баланс, укажите окислитель и восстановитель в этой реакции.

31. Из предложенного перечня веществ выберите вещества, между которыми может протекать реакция ионного обмена. Запишите молекулярное, полное и сокращенное ионное уравнения только одной из возможных реакций.

32.

33.

34.

35.

ЕГЭ по химии – экзамен, который сдают выпускники, планирующие поступать в ВУЗ на определенные специальности, связанные с данной дисциплиной. Химия не входит в перечень обязательных предметов, по статистике, из 10 выпускников химию сдает 1.

  • На тестирование и выполнение всех заданий выпускник получает 3 часа времени – планирование и распределение времени на работу со всеми заданиям является важной задачей испытуемого.
  • Обычно экзамен включает 35-40 заданий, которые делятся на 2 логических блока.
  • Как и остальные ЕГЭ, испытание по химии делится на 2 логических блока: тестирование (выбор правильного варианта или вариантов из предложенных) и вопросы, на которые требуется дать развернутые ответы. Именно второй блок обычно занимает больше времени, поэтому испытуемому необходимо рационально распределять время.

  • Главное – иметь надежные, глубокие теоретические знания, которые помогут успешно выполнять различные задания первого и второго блоков.
  • Готовиться нужно начинать заранее, чтобы систематически проработать все темы – полугода может быть мало. Лучший вариант – начать подготовку еще в 10-ом классе.
  • Определите темы, которые составляют для вас наибольшие проблемы, чтобы, обращаясь за помощью к учителю или репетитору, знать, что спрашивать.
  • Учиться выполнять задания, типичные для ЕГЭ по химии – мало владеть теорией, необходимо довести навыки выполнения задач и различных заданий до автоматизма.
Полезные советы: как сдать ЕГЭ по химии?
  • Не всегда самостоятельная подготовка эффективна, поэтому стоит найти специалиста, к которому вы сможете обратиться за помощью. Лучший вариант – профессиональный репетитор. Также не стоит бояться задавать вопросы школьному учителю. Не пренебрегайте школьным образованием, внимательно выполняйте задания на уроках!
  • На экзамене есть подсказки! Главное – научиться пользоваться этими источниками информации. Ученик располагает таблицей Менделеева, таблицами напряжения металлов и растворимости – это около 70% данных, которые помогут разобраться в различных заданиях.
Как работать с таблицами? Главное – внимательно изучить особенности элементов, научиться «читать» таблицу. Основные данные об элементах: валентность, строение атомов, свойства, уровень окисления.
  • Химия требует основательных знаний в математике – без этого будет трудно решать задачи. Обязательно повторите работу с процентами и пропорциями.
  • Выучите формулы, которые необходимы для решения задач по химии.
  • Изучите теорию: пригодятся учебники, справочники, сборники задач.
  • Оптимальный способ закрепить теоретические задания – активно решать задания по химии. В онлайн режиме вы можете решать в любом количестве, совершенствовать навыки решения задач разного типа и уровня сложности.
  • Спорные моменты в заданиях и ошибки рекомендуется разбирать и анализировать при помощи учителя или репетитора.
«Решу ЕГЭ по химии» – это возможность каждого ученика, который планирует сдавать этот предмет, проверять уровень свои знаний, восполнять пробелы, в итоге – получить высокий балл и поступить в ВУЗ.

Уважаемые выпускники и абитуриенты!

Настоящее учебное пособие представляет собой сборник заданий для подготовки к сдаче Единого государственного экзамена (ЕГЭ) по химии, который является как выпускным экзаменом за курс средней школы, так и вступительным экзаменом в вуз. Структура пособия отражает современные требования к процедуре сдачи ЕГЭ по химии, что позволит вам лучше подготовиться к новым формам выпускной аттестации и к поступлению в вузы.

Пособие состоит из 10 вариантов заданий, которые по форме и содержанию приближены к демоверсии ЕГЭ и не выходят за рамки содержания курса химии, нормативно определенного Федеральным компонентом государственного стандарта общего образования. Химия (приказ Минобразования № 1089 от 05.03.2004 г.).

Уровень предъявления содержания учебного материала в заданиях соотнесен с требованиями государственного стандарта к подготовке выпускников средней (полной) школы по химии.

В контрольных измерительных» материалах Единого государственного экзамена используются задания трех типов:

Задания базового уровня сложности с кратким ответом,

Задания повышенного уровня сложности с кратким ответом,

Задания высокого уровня сложности с развернутым ответом.

Каждый вариант экзаменационной работы построен по единому плану. Работа состоит из двух частей, включающих в себя суммарно 34 задания. Часть 1 содержит 29 заданий с кратким ответом базового и повышенного уровня сложности. Часть 2 содержит 5 заданий высокого уровня сложности, с развернутым ответом (задания под номерами 30-34).

В заданиях высокого уровня сложности текст решения записывается на специальном бланке. Задания именно этого типа составляют основную часть письменной работы по химии на вступительных экзаменах в вузы.

Задания с развернутым ответом могут быть выполнены выпускниками разными способами. Поэтому решения, приведенные в методических рекомендациях для экзаменаторов (имеются в виду критерии оценивания заданий части 2 в разделе «Решение заданий варианта 1»), следует рассматривать как один из возможных вариантов ответов.

Назначение данного пособия — ознакомить читателей со структурой контрольных измерительных материалов, числом, формой и уровнем сложности заданий. Эти сведения позволят выпускникам выработать стратегию подготовки и сдачи ЕГЭ в соответствии с целями, которые они ставят перед собой.

В пособии даны ответы к заданиям всех вариантов и приведены подробные решения всех заданий десятого варианта. Кроме того, приведены образцы бланков, используемых на ЕГЭ для записи ответов и решений. Именно такой вид имеют контрольные измерительные материалы, которые получают выпускники на экзамене. Прежде чем приступить к решению заданий, изучите внимательно все инструкции.

Настоящее пособие адресовано учащимся-старшеклассникам и абитуриентам для самоподготовки и самоконтроля. Пособие может быть использовано учителями химии и методистами для подготовки учащихся к итоговой аттестации по химии за курс средней школы, причем как в форме ЕГЭ, так и традиционного письменного экзамена.

Разработали тренировочные тесты по химии для ЕГЭ 2020 с ответами и решениями.

При подготовке изучите 10 тренировочных вариантов, составленные на основе новой демоверсии .

Особенности заданий в тестах ЕГЭ по химии

Рассмотрим типологию и структуру некоторых заданий первой части:

  • – в условии дан ряд химических элементов и вопросы касательно каждого из них, обратите внимание на количество клеточек для ответа – их две, следовательно, и вариантов решения два;
  • – соответствие между двумя множествами: будут два столбца, в одном формулы веществ, а во втором – группа веществ, необходимо будет найти соответствия.
  • В первой части будут еще и задачи, требующие поведения «мысленного химического эксперимента», при котором ученик выбирает формулы, позволяющие найти правильный ответ на экзаменационный вопрос.
  • Задачи второго блока выше по уровню сложности и требуют владения несколькими элементами содержания и несколькими навыками и умениями.

Подсказка : при решении задачи важно определить класс, группу вещества и свойства.

Задания с развернутыми ответами ориентированы на проверку знаний по основным курсам:

  • Строение атома;
  • Периодические законы;
  • Неорганическая химия;
  • Органическая химия;
  • Расчеты по формулам;
  • Применение химии в жизни.

Подготовка к ЕГЭ по химии – быстро и качественно

Быстро – значит, не менее, чем за полгода:

  1. Подтянуть математику.
  2. Повторить всю теорию.
  3. Решать онлайн пробные варианты по химии, смотреть видеоуроки.

Наш сайт предусмотрел такую возможность – заходи тренируйся и получай высокие баллы на экзаменах.

Варианты егэ химия на каждую неделю. Демонстрационные варианты ЕГЭ по химии (11 класс). Тренировочные варианты ЕГЭ по химии

Нитрид натрия массой 8,3 г прореагировал с серной кислотой с массовой долей 20% и массой 490 г. Затем к полученному раствору добавили кристаллическую соду массой 57,2 г. Найдитее массовую долю (%) кислоты в конечном растворе. Записывайте уравнения реакций, которые указаны в условии задачи, приводите все необходимые вычисления (указывайте единицы измерения искомых физических величин). Ответ для сайта округлите до целого числа.

Реальное ЕГЭ 2017. Задание 34.

Циклическое вещество А (не содержит кислорода и заместителей) окисляется с разрывом цикла до вещества Б массой 20,8 г, продуктами горения которого являетя углекислый газ объёмом 13,44 л и вода массой 7,2 г. На основании данных условия задания: 1) произведите вычисления, необходимые для установления молекулярной формулы органического вещества Б; 2) запишите молекулярные формулы органических веществ А и Б; 3) составьте структурные формулы органических веществ А и Б, которые однозначно отражают порядок связи атомов в молекуле; 4) напишите уравнение реакции окисления вещества А сернокислым раствором перманганата калия с образованием вещества Б. В ответе для сайта укажите сумму всех атомов в одной молекуле исходного органического вещества А.

ЕГЭ по химии – экзамен, который сдают выпускники, планирующие поступать в ВУЗ на определенные специальности, связанные с данной дисциплиной. Химия не входит в перечень обязательных предметов, по статистике, из 10 выпускников химию сдает 1.

  • На тестирование и выполнение всех заданий выпускник получает 3 часа времени – планирование и распределение времени на работу со всеми заданиям является важной задачей испытуемого.
  • Обычно экзамен включает 35-40 заданий, которые делятся на 2 логических блока.
  • Как и остальные ЕГЭ, испытание по химии делится на 2 логических блока: тестирование (выбор правильного варианта или вариантов из предложенных) и вопросы, на которые требуется дать развернутые ответы. Именно второй блок обычно занимает больше времени, поэтому испытуемому необходимо рационально распределять время.

  • Главное – иметь надежные, глубокие теоретические знания, которые помогут успешно выполнять различные задания первого и второго блоков.
  • Готовиться нужно начинать заранее, чтобы систематически проработать все темы – полугода может быть мало. Лучший вариант – начать подготовку еще в 10-ом классе.
  • Определите темы, которые составляют для вас наибольшие проблемы, чтобы, обращаясь за помощью к учителю или репетитору, знать, что спрашивать.
  • Учиться выполнять задания, типичные для ЕГЭ по химии – мало владеть теорией, необходимо довести навыки выполнения задач и различных заданий до автоматизма.
Полезные советы: как сдать ЕГЭ по химии?
  • Не всегда самостоятельная подготовка эффективна, поэтому стоит найти специалиста, к которому вы сможете обратиться за помощью. Лучший вариант – профессиональный репетитор. Также не стоит бояться задавать вопросы школьному учителю. Не пренебрегайте школьным образованием, внимательно выполняйте задания на уроках!
  • На экзамене есть подсказки! Главное – научиться пользоваться этими источниками информации. Ученик располагает таблицей Менделеева, таблицами напряжения металлов и растворимости – это около 70% данных, которые помогут разобраться в различных заданиях.
Как работать с таблицами? Главное – внимательно изучить особенности элементов, научиться «читать» таблицу. Основные данные об элементах: валентность, строение атомов, свойства, уровень окисления.
  • Химия требует основательных знаний в математике – без этого будет трудно решать задачи. Обязательно повторите работу с процентами и пропорциями.
  • Выучите формулы, которые необходимы для решения задач по химии.
  • Изучите теорию: пригодятся учебники, справочники, сборники задач.
  • Оптимальный способ закрепить теоретические задания – активно решать задания по химии. В онлайн режиме вы можете решать в любом количестве, совершенствовать навыки решения задач разного типа и уровня сложности.
  • Спорные моменты в заданиях и ошибки рекомендуется разбирать и анализировать при помощи учителя или репетитора.
«Решу ЕГЭ по химии» – это возможность каждого ученика, который планирует сдавать этот предмет, проверять уровень свои знаний, восполнять пробелы, в итоге – получить высокий балл и поступить в ВУЗ.

Демонстрационные варианты ЕГЭ по химии для 11 класса состоят из двух частей. В первую часть входят задания, к которым нужно дать краткий ответ. К заданиям из второй части необходимо дать развернутый ответ.

Все демонстрационные варианты ЕГЭ по химии содержат верные ответы ко всем заданиям и критерии оценивания для заданий с развернутым ответом.

В по сравнению с изменений нет.

Демонстрационные варианты ЕГЭ по химии

Отметим, что демонстрационные варианты по химии представлены в формате pdf, и для их просмотра необходимо, чтобы на Вашем компьютере был установлен, например, свободно распространяемый программный пакет Adobe Reader.

Демонстрационный вариант ЕГЭ по химии за 2007 год
Демонстрационный вариант ЕГЭ по химии за 2002 год
Демонстрационный вариант ЕГЭ по химии за 2004 год
Демонстрационный вариант ЕГЭ по химии за 2005 год
Демонстрационный вариант ЕГЭ по химии за 2006 год
Демонстрационный вариант ЕГЭ по химии за 2008 год
Демонстрационный вариант ЕГЭ по химии за 2009 год
Демонстрационный вариант ЕГЭ по химии за 2010 год
Демонстрационный вариант ЕГЭ по химии за 2011 год
Демонстрационный вариант ЕГЭ по химии за 2012 год
Демонстрационный вариант ЕГЭ по химии за 2013 год
Демонстрационный вариант ЕГЭ по химии за 2014 год
Демонстрационный вариант ЕГЭ по химии за 2015 год
Демонстрационный вариант ЕГЭ по химии за 2016 год
Демонстрационный вариант ЕГЭ по химии за 2017 год
Демонстрационный вариант ЕГЭ по химии за 2018 год
Демонстрационный вариант ЕГЭ по химии за 2019 год

Изменения в демонстрационных вариантах ЕГЭ по химии

Демонстрационные варианты ЕГЭ по химии для 11 класса за 2002 — 2014 годы состояли из трех частей. Первая часть включала в себя задания, в которых нужно выбрать один из предложенных ответов. К заданиям из второй части требовалось дать краткий ответ. К заданиям из третьей части нужно было дать развернутый ответ.

В 2014 году в демонстрационный вариант ЕГЭ по химии были внесены следующие изменения :

  • все расчетные задачи , выполнение которых оценивалось в 1 балл, были помещены в часть 1 работы (А26–А28) ,
  • тема «Реакции окислительно-восстановительные» проверялась с помощью заданий В2 и С1 ;
  • тема «Гидролиз солей» проверялась только с помощью задания В4 ;
  • было включено новое задание (на позиции В6 ) для проверки тем «качественные реакции на неорганические вещества и ионы», «качественные реакции органических соединений»
  • общее количество заданий в каждом варианте стало 42 (вместо 43 в работе 2013 г.).

В 2015 году в были внесены принципиальные изменения :

    Вариант стал состоять из двух частей (часть 1 — задания с кратким ответом , часть 2 — задания с развернутым ответом ).

    Нумерация заданий стала сквозной по всему варианту без буквенных обозначений А, В, С.

    Была изменена форма записи ответа в заданиях с выбором ответа: ответ стало нужно записывать цифрой с номером правильного ответа (а не отмечать крестиком).

    Было уменьшено число заданий базового уровня сложности с 28 до 26 заданий .

    Максимальный балл за выполнение всех заданий экзаменационной работы 2015 года стал 64 (вместо 65 баллов в 2014 году).

  • Была изменена система оценивания задания на нахождение молекулярной формулы вещества . Максимальный балл за его выполнение – 4 (вместо 3 баллов в 2014 году).

В 2016 году в демонстрационный вариант по химии внесены существенные изменения по сравнению с предыдущим 2015 годом:

    В части 1 изменен формат заданий 6, 11, 18, 24, 25 и 26 базового уровня сложности с кратким ответом.

    Изменен формат заданий 34 и 35 повышенного уровня сложности: в этих заданиях теперь требуется установить соответствие вместо выбора нескольких правильных ответов из предложенного списка.

    Изменено распределение заданий по уровню сложности и видам проверяемых умений.

В 2017 году в по сравнению с демонстрационным вариантом 2016 года по химии произошли существенные изменения. Была оптимизирована структура экзаменационной работы:

    Была изменена структура первой части демонтрационного варианта: из него были исключены задания с выбором одного ответа; задания были сгруппированы по отдельным тематическим блокам, каждый из которых стал содержать задания как базового, так и повышенного уровня сложности..

    Было уменьшено общее количество заданий до 34.

    Была изменена шкала оценивания (с 1 до 2 баллов) выполнения заданий базового уровня сложности, которые проверяют усвоение знаний о генетической связи неорганических и органических веществ (9 и 17).

    Максимальный балл за выполнение всех заданий экзаменационной работы был уменьшен до 60 баллов .

В 2018 году в демонстрационном варианте ЕГЭ по химии по сравнению с демонстрационным вариантом 2017 года по химии произошли следующие изменения :

    Было добавлено задание 30 высокого уровня сложности с развернутым ответом,

    Максимальный балл за выполнение всех заданий экзаменационной работы остался без изменения за счет изменения шкалы оценивания заданий части 1.

В демонстрационном варианте ЕГЭ 2019 года по химии по сравнению с демонстрационным вариантом 2018 года по химии изменений не было.

На нашем сайте можно также ознакомиться с подготовленными преподавателями нашего учебного центра «Резольвента» учебными материалами для подготовки к ЕГЭ по математике .

Для школьников 10 и 11 классов, желающих хорошо подготовиться и сдать ЕГЭ по математике или русскому языку на высокий балл, учебный центр «Резольвента» проводит

У нас также для школьников организованы

Тренировочные варианты ЕГЭ по химии

Разработали тренировочные тесты по химии для ЕГЭ 2019 с ответами и решениями.

При подготовке изучите 10 тренировочных вариантов, составленные на основе новой .

Особенности заданий в тестах ЕГЭ по химии

Рассмотрим типологию и структуру некоторых заданий первой части:

  • – в условии дан ряд химических элементов и вопросы касательно каждого из них, обратите внимание на количество клеточек для ответа – их две, следовательно, и вариантов решения два;
  • – соответствие между двумя множествами: будут два столбца, в одном формулы веществ, а во втором – группа веществ, необходимо будет найти соответствия.
  • В первой части будут еще и задачи, требующие поведения «мысленного химического эксперимента», при котором ученик выбирает формулы, позволяющие найти правильный ответ на экзаменационный вопрос.
  • Задачи второго блока выше по уровню сложности и требуют владения несколькими элементами содержания и несколькими навыками и умениями.

Подсказка : при решении задачи важно определить класс, группу вещества и свойства.

Задания с развернутыми ответами ориентированы на проверку знаний по основным курсам:


Подготовка к ЕГЭ по химии – быстро и качественно

Быстро – значит, не менее, чем за полгода:

  1. Подтянуть математику.
  2. Повторить всю теорию.
  3. Решать онлайн пробные варианты по химии, смотреть видеоуроки.

Наш сайт предусмотрел такую возможность – заходи тренируйся и получай высокие баллы на экзаменах.

Возбужденному состоянию атома соответствует электронная конфигурация

1) 1s 2 2s 2 2p 6 3s 1

2) 1s 2 2s 2 2p 6 3s 2 3p 6

3) 1s 2 2s 2 2p 6 3s 1 3p 2

Ответ: 3

Пояснение:

Энергия 3s-подуровня ниже энергии 3p-подуровня, однако 3s-подуровень, на котором должно находиться 2 электрона, заполнен не полностью. Следовательно, такая электронная конфигурация соответствует возбужденному состоянию атома (алюминия).

Четвертый вариант не является ответом в связи с тем, что, хотя 3d-уровень и не заполнен, но его энергия выше 4s-подуровня, т.е. в данном случает он заполняется последним.

В каком ряду химические элементы расположены в порядке уменьшения их атомного радиуса?

1) Rb → K → Na

2) Mg → Ca → Sr

3) Si → Al → Mg

Ответ: 1

Пояснение:

Атомный радиус элементов уменьшается при уменьшении числа электронных оболочек (число электронных оболочек соответствует номеру периода Периодической системы химических элементов) и при переходе к неметаллам (т.е. при увеличении числа электронов на внешнем уровне). Следовательно, в таблице химических элементов атомный радиус элементов уменьшается снизу вверх и слева направо.

Между атомами с одинаковой относительной электроотрицательностью образуется химическая связь

2) ковалентная полярная

3) ковалентная неполярная

Ответ: 3

Пояснение:

Между атомами с одинаковой относительной электроотрицательностью образуется ковалентная неполярная связь, так как не происходит смещение электронной плотности.

Степени окисления серы и азота в (NH 4) 2 SO 3 соответственно равны

1) +4 и -3 2) -2 и +5 3) +6 и +3 4) -2 и +4

Ответ: 1

Пояснение:

(NH 4) 2 SO 3 (сульфит аммония) – соль, образованная сернистой кислотой и аммиаком, следовательно, степени окисления серы и азота равны +4 и -3 соответственно (степень окисления серы в сернистой кислоте +4, степень окисления азота в аммиаке -3).

Атомную кристаллическую решетку имеет

1) белый фосфор

3) кремний

Ответ: 3

Пояснение:

Белый фосфор имеет молекулярную кристаллическую решетку, формула молекулы белого фосфора – P 4 .

Обе аллотропные модификации серы (ромбическая и моноклинная) имеют молекулярные кристаллические решетки, в узлах которых находятся циклические коронообразные молекулы S 8 .

Свинец является металлом и имеет металлическую кристаллическую решетку.

Кремний имеет кристаллическую решетку типа алмаза, однако ввиду большей длины связи Si-Si по сравнению C-C уступает алмазу по твердости.

Среди перечисленных веществ выберите три вещества, которые относятся к амфотерным гидроксидам.

Ответ: 245

Пояснение:

К амфотерным металлам относятся Be, Zn, Al (можно запомнить «БеЗнАл»), а также Fe III и Cr III . Следовательно, из предложенных вариантов ответов к амфотерным гидроксидам относятся Be(OH) 2 , Zn(OH) 2 , Fe(OH) 3 .

Соединение Al(OH) 2 Br является основной солью.

Верны ли следующие суждения о свойствах азота?

А. При обычных условиях азот реагирует с серебром.

Б. Азот при обычных условиях в отсутствие катализатора не реагирует с водородом.

1) верно только А

2) верно только Б

3) верны оба суждения

Ответ: 2

Пояснение:

Азот является весьма инертным газом и с металлами, кроме лития, при обычных условиях не реагирует.

Взаимодействие азота с водородом относится к промышленному получению аммиака. Процесс является экзотермическим обратимым и протекает только в присутствии катализаторов.

Оксид углерода (IV) реагирует с каждым из двух веществ:

1) кислородом и водой

2) водой и оксидом кальция

3) сульфатом калия и гидроксидом натрия

4) оксидом кремния (IV) и водородом

Ответ: 2

Пояснение:

Оксид углерода (IV) (углекислый газ) является кислотным оксидом, следовательно, взаимодействует с водой с образованием неустойчивой угольной кислоты, щелочами и оксидами щелочных и щелочноземельных металлов с образованием солей:

CO 2 + H 2 O ↔ H 2 CO 3

CO 2 + CaO → CaCO 3

С раствором гидроксида натрия реагирует каждое из двух

3) H 2 O и P 2 O 5

Ответ: 4

Пояснение:

NaOH является щелочью (обладает основными свойствами), следовательно, возможно взаимодействие с кислотным оксидом − SO 2 и гидроксидом амфотерного металла − Al(OH) 3:

2NaOH + SO 2 → Na 2 SO 3 + H 2 O или NaOH + SO 2 → NaHSO 3

NaOH + Al(OH) 3 → Na

Карбонат кальция взаимодействует с раствором

1) гидроксида натрия

2) хлороводорода

3) хлорида бария

Ответ: 2

Пояснение:

Карбонат кальция – не растворимая в воде соль, следовательно, не взаимодействует с солями и основаниями. Карбонат кальция растворяется в сильных кислотах с образованием солей и выделением углекислого газа:

CaCO 3 + 2HCl → CaCl 2 + CO 2 + H 2 O

В схеме превращений

1) оксид железа (II)

2) гидроксид железа (III)

3) гидроксид железа (II)

4) хлорид железа (II)

Ответ: X-5; Y-2

Пояснение:

Хлор является сильным окислителем (окислительная способность галогенов увеличивает от I 2 к F 2), окисляет железо до Fe +3:

2Fe + 3Cl 2 → 2FeCl 3

Хлорид железа (III) является растворимой солью и вступает в обменные реакции с щелочами с образованием осадка – гидроксида железа (III):

FeCl 3 + 3NaOH → Fe(OH) 3 ↓ + NaCl

Гомологами являются

1) глицерин и этиленгликоль

2) метанол и бутанол-1

3) пропин и этилен

Ответ: 2

Пояснение:

Гомологи – вещества, принадлежащие одному классу органических соединений и отличающиеся на одну или несколько CH 2 -групп.

Глицерин и этиленгликоль являются трехатомным и двухатомным спиртами соответственно, отличаются количеством атомов кислорода, поэтому не являются ни изомерами, ни гомологами.
Метанол и бутанол-1 являются первичными спиртами с неразветвленным скелетом, отличаются на две CH 2 -группы, следовательно, являются гомолоами.

Пропин и этилен относятся к классам алкинов и алкенов соответственно, содержат разное количество атомов углерода и водорода, следовательно, не являются между собой ни гомологами, ни изомерами.

Пропанон и пропаналь принадлежат к разным классам органических соединений, но содержат по 3 атома углерода, 6 атомов водорода и по 1 атому кислорода, следовательно, являются изомерами по функциональной группе.

Для бутена-2 невозможна реакция

1) дегидратации

2) полимеризации

3) галогенирования

Ответ: 1

Пояснение:

Бутен-2 относится к классу алкенов, вступает в реакции присоединения с галогенами, галогеноводородами, водой и водородом. Кроме того, непредельные углеводороды полимеризуются.

Реакция дегидратации – это реакция, протекающая с отщеплением молекулы воды. Поскольку бутен-2 является углеводородом, т.е. не содержит гетероатомов, отщепление воды невозможно.

Фенол не взаимодействует с

1) азотной кислотой

2) гидроксидом натрия

3) бромной водой

Ответ: 4

Пояснение:

С фенолом в реакции электрофильного замещения по бензольному кольцу вступают азотная кислота и бромная вода, в результате чего образуются нитрофенол и бромфенол соответственно.

Фенол, обладающий слабыми кислотными свойствами, вступает в реакции со щелочами с образованием фенолятов. В данном случает образуется фенолят натрия.

Алканы с фенолом не реагируют.

Метиловый эфир уксусной кислоты вступает в реакцию с

1) NaCl 2) Br 2 (р-р) 3) Cu(OH) 2 4) NaOH(р-р)

Ответ: 4

Пояснение:

Метиловый эфир уксусной кислоты (метилацетат) принадлежит к классу сложных эфиров, подвергается кислотному и щелочному гидролизу. В условиях кислотного гидролиза метилацетат превращается в уксусную кислоту и метанол, в условиях щелочного гидролиза с гидроксидом натрия – ацетат натрия и метанол.

Бутен-2 можно получить дегидратацией

1) бутанона 2) бутанола-1 3) бутанола-2 4) бутаналя

Ответ: 3

Пояснение:

Одним из способов получения алкенов является реакция внутримолекулярной дегидратации первичных и вторичных спиртов, протекающая в присутствии безводной серной кислоты и при температуре выше 140 o C. Отщепление молекулы воды от молекулы спирта протекает по правилу Зайцева: атом водорода и гидроксильная группа отщепляются от соседних атомов углерода, причем водород отщепляется от того атома углерода, при котором находится наименьшее число атомов водорода. Таким образом, внутримолекулярная дегидратация первичного спирта − бутанола-1 приводит к образованию бутена-1, внутримолекулярная дегидратация вторичного спирта – бутанола-2 к образованию бутена-2.

Метиламин может реагировать со (с)

1) щелочами и спиртами

2) щелочами и кислотами

3) кислородом и щелочами

4) кислотами и кислородом

Ответ: 4

Пояснение:

Метиламин принадлежит к классу аминов и обладает ввиду наличия на атоме азота неподеленной электронной пары основными свойствами. Кроме того, основные свойства метиламина выражены сильнее, чем у аммиака, благодаря наличию метильной группы, обладающей положительным индуктивным эффектом. Таким образом, обладая основными свойствами, метиламин взаимодействует с кислотами с образованием солей. В атмосфере кислорода метиламин сгорает до углекислого газа, азота и воды.

В заданной схеме превращений

веществами X и Y соответственно являются

1) этандиол-1,2

3) ацетилен

4) диэтиловый эфир

Ответ: X-2; Y-5

Пояснение:

Бромэтан в водном растворе щелочи вступает в реакцию нуклеофильного замещения с образованием этанола:

CH 3 -CH 2 -Br + NaOH(водн.) → CH 3 -CH 2 -OH + NaBr

В условиях концентрированной серной кислоты при температуре выше 140 0 C протекает внутримолекулярная дегидратация с образованием этилена и воды:

Все алкены легко вступают в реакцию присоединяется с бромом:

CH 2 =CH 2 + Br 2 → CH 2 Br-CH 2 Br

К реакциям замещения относится взаимодействие

1) ацетилена и бромоводорода

2) пропана и хлора

3) этена и хлора

4) этилена и хлороводорода

Ответ: 2

Пояснение:

К реакциям присоединения относятся взаимодействие непредельных углеводородов (алкенов, алкинов, алкадиенов) с галогенами, галогеноводородами, водородом и водой. Ацетилен (этин) и этилен принадлежат классам алкинов и алкенов соответственно, следовательно, с бромоводородом, хлороводородом и хлором вступают в реакции присоединения.

В реакции замещения с галогенами на свету или при повышенной температуре вступают алканы. Реакция протекает по цепному механизму с участием свободных радикалов – частиц с одним неспаренным электроном:

На скорость химической реакции

HCOOCH 3(ж) + H 2 O (ж) → HCOOH (ж) + CH 3 OH (ж)

не оказывает влияния

1) повышение давления

2) повышение температуры

3) изменение концентрации HCOOCH 3

4) использование катализатора

Ответ: 1

Пояснение:

На скорость реакции оказывают влияние изменение температуры и концентраций исходных реагентов, а также использование катализатора. Согласно эмпирическому правилу Вант-Гоффа, при повышении температуры на каждые 10 градусов константа скорости гомогенной реакции увеличивается в 2-4 раза.

Использование катализатора также ускоряет реакции, при этом катализатор в состав продуктов не входит.

Исходные вещества и продукты реакции находятся в жидкой фазе, следовательно, изменение давления не сказывается на скорости данной реакции.

Сокращенное ионное уравнение

Fe +3 + 3OH − = Fe(OH) 3 ↓

соответствует молекулярному уравнению реакции

1) FeCl 3 + 3NaOH = Fe(OH) 3 ↓ + 3NaCl

2) 4Fe(OH) 2 + O 2 + 2H 2 O = 4Fe(OH) 3 ↓

3) FeCl 3 + 3NaHCO 3 = Fe(OH) 3 ↓ + 3CO 2 + 3NaCl

Ответ: 1

Пояснение:

В водном растворе на ионы диссоциируют растворимые соли, щелочи и сильные кислоты, в молекулярном виде записываются нерастворимые основания, нерастворимые соли, слабые кислоты, газы, простые вещества.

Условию растворимости солей и оснований соответствует первое уравнение, в котором соль вступает в обменную реакцию со щелочью с образованием нерастворимого основания и другой растворимой соли.

Полное ионное уравнение записывается в следующем виде:

Fe +3 + 3Cl − + 3Na + + 3OH − = Fe(OH) 3 ↓ + 3Cl − + 3Na +

Какой из перечисленных газов токсичен и имеет резкий запах?

1) водород

2) оксид углерода (II)

Ответ: 3

Пояснение:

Водород и углекислый газ – нетоксичные газы без запаха. Угарный газ и хлор являются токсичными, однако в отличие от CO хлор обладает резким запахом.

В реакцию полимеризации вступает

1) фенол 2) бензол 3) толуол 4) стирол

Ответ: 4

Пояснение:

Все вещества из предложенных вариантов являются ароматическими углеводородами, но для ароматических систем реакции полимеризации не характерны. В молекуле стирола содержится винил-радикал, являющийся фрагментом молекулы этилена, для которого характерны реакции полимеризации. Таким образом, стирол полимеризуется с образованием полистирола.

К 240 г раствора с массовой долей соли 10% добавили 160 мл воды. Определите массовую долю соли в полученном растворе. (Запишите число с точностью до целых.)

Ответ: 6% Пояснение:

Массовая доля соли в растворе рассчитывается по формуле:

Исходя из этой формулы, вычислим массу соли в исходном растворе:

m(в-ва) = ω(в-ва в исх. р-ре) . m(исх. р-ра)/100% = 10% . 240 г/100% = 24 г

При добавлении воды в раствор масса полученного раствора составит 160 г + 240 г = 400 г (плотность воды 1 г/мл).

Массовая доля соли в полученном растворе составит:

Рассчитайте, какой объем азота (н.у.) образуется при полном сгорании 67,2 л (н.у.) аммиака. (Запишите число с точностью до десятых.)

Ответ: 33,6 л

Пояснение:

Полное сгорание аммиака в кислороде описывается уравнением:

4NH 3 + 3O 2 → 2N 2 + 6H 2 O

Следствием из закона Авогадро является то, что объемы газов, находящихся в одинаковых условиях, относятся друг к другу так же, как и количества молей этих газов. Таким образом, по уравнению реакции

ν(N 2) = 1/2ν(NH 3),

следовательно, объемы аммиака и азота соотносятся между собой точно так же:

V(N 2) = 1/2V(NH 3)

V(N 2) = 1/2V(NH 3) = 67,2 л/2 = 33,6 л

Какой объем (в литрах при н.у.) кислорода образуется при разложении 4 моль пероксида водорода? (Запишите число с точностью до десятых).

Ответ: 44,8 л

Пояснение:

В присутствии катализатора − диоксида марганца перекись разлагается с образованием кислорода и воды:

2H 2 O 2 → 2H 2 O + O 2

По уравнению реакции количество образующегося кислорода в два раза меньше количества пероксида водорода:

ν (O 2) = 1/2ν (H 2 O 2), следовательно, ν (O 2) = 4 моль/2 = 2 моль.

Объем газов рассчитывается по формуле:

V = V m · ν , где V m – молярный объем газов при н.у., равный 22,4 л/моль

Объем кислорода, образующийся при разложении перекиси, равен:

V(O 2) = V m · ν (O 2) = 22,4 л/моль ·2 моль = 44,8 л

Установите соответствие между классами соединений и тривиальным названием вещества, которое является его представителем.

Ответ: А-3; Б-2; В-1; Г-5

Пояснение:

Спирты – органические вещества, содержащие одну или несколько гидроксильных групп (-OH), непосредственно связанных с насыщенным атомом углерода. Этиленгликоль является двухатомным спиртом, содержит две гидроксильные группы: CH 2 (OH)-CH 2 OH.

Углеводы – органические вещества, содержащие карбонильную и несколько гидроксильных групп, общая формула углеводов записывается в виде C n (H 2 O) m (где m, n > 3). Из предложенных вариантов к углеводам относится крахмал – полисахарид, высокомолекулярный углевод, состоящий из большого числа остатков моносахаридов, формула которого записывается в виде (C 6 H 10 O 5) n .

Углеводороды – органические вещества, в состав которых входят исключительно два элемента – углерод и водород. К углеводородам из предложенных вариантов относится толуол – ароматическое соединение, состоящее только из атомов углерода и водорода и не содержащее функциональных групп с гетероатомами.

Карбоновые кислоты – органические вещества, в молекулах которых содержится карбоксильная группа, состоящая из связанных между собой карбонильной и гидроксильной групп. К классу карбоновых кислот относится масляная (бутановая) кислота – C 3 H 7 COOH.

Установите соответствие между уравнением реакции и изменением степени окисления окислителя в ней.

УРАВНЕНИЕ РЕАКЦИИ

А) 4NH 3 + 5O 2 = 4NO + 6H 2 O

Б) 2Cu(NO 3) 2 = 2CuO + 4NO 2 + O 2

В) 4Zn + 10HNO 3 = NH 4 NO 3 + 4Zn(NO 3) 2 + 3H 2 O

Г) 3NO 2 + H 2 O = 2HNO 3 + NO

ИЗМЕНЕНИЕ СТЕПЕНИ ОКИСЛЕНИЯ ОКИСЛИТЕЛЯ

Ответ: А-1; Б-4; В-6; Г-3

Пояснение:

Окислитель – вещество, в состав которого входят атомы, способные в ходе химической реакции присоединять электроны и таким образом понижать степень окисления.

Восстановитель – вещество, в состав которого входят атомы, способные в ходе химической реакции отдавать электроны и таким образом повышать степень окисления.

А) Окисление аммиака кислородом в присутствии катализатора приводит к образованию монооксида азота и воды. Окислителем является молекулярный кислород, изначально имеющий степень окисления 0, который, присоединяя электроны, восстанавливается до степени окисления -2 в соединениях NO и H 2 O.

Б) Нитрат меди Cu(NO 3) 2 – соль, содержащая кислотный остаток азотной кислотой. Степени окисления азота и кислорода в нитрат-анионе равны +5 и -2 соответственно. В ходе реакции нитрат-анион превращается в диоксид азота NO 2 (со степенью окисления азота +4) и кислород O 2 (со степенью окисления 0). Следовательно, окислителем является азот, поскольку понижает степень окисления с +5 в нитрат-ионе до +4 в диоксиде азота.

В) В данной окислительно-восстановительной реакции окислителем является азотная кислота, которая, превращаясь в нитрат аммония, понижает степень окисления азота с +5 (в азотной кислоте) до -3 (в катионе аммония). Степень окисления азота в кислотных остатках нитрата аммония и нитрата цинка остается неизменной, т.е. такая же, как у азота в HNO 3 .

Г) В данной реакции азот в диоксиде диспропорционирует, т.е. одновременно и повышает (от N +4 в NO 2 до N +5 в HNO 3), и понижает (от N +4 в NO 2 до N +2 в NO) свою степень окисления.

Установите соответствие между формулой вещества и продуктами электролиза его водного раствора, которые выделились на инертных электродах.

Ответ: А-4; Б-3; В-2; Г-5

Пояснение:

Электролиз – окислительно-восстановительный процесс, протекающий на электродах при прохождении постоянного электрического тока через раствор или расплав электролита. На катоде преимущественно происходит восстановление тех катионов, которые обладают наибольшей окислительной активностью. На аноде в первую очередь окисляются те анионы, которые обладают наибольшей восстановительной способностью.

Электролиз водного раствора

1) Процесс электролиза водных растворов на катоде не зависит от материала катода, но зависит от положения катиона металла в электрохимическом ряду напряжений.

Для катионов в ряду

Li + − Al 3+ процесс восстановления:

2H 2 O + 2e → H 2 + 2OH − (на катоде выделяется H 2)

Zn 2+ − Pb 2+ процесс восстановления:

Me n + + ne → Me 0 и 2H 2 O + 2e → H 2 + 2OH − (на катоде выделяется H 2 и Me)

Cu 2+ − Au 3+ процесс восстановления Me n + + ne → Me 0 (на катоде выделяется Me)

2) Процесс электролиза водных растворов на аноде зависит от материала анода и от природы аниона. Если анод нерастворимый,т.е. инертный (платина, золото, уголь, графит), то процесс будет зависеть только от природы анионов.

Для анионов F − , SO 4 2- , NO 3 − , PO 4 3- , OH − процесс окисления:

4OH − − 4e → O 2 + 2H 2 O или 2H 2 O – 4e → O 2 + 4H + (на аноде выделяется кислород)

галогенид-ионов (кроме F −) процесс окисления 2Hal − − 2e → Hal 2 (выделяются свободные галогены)

органических кислот процесс окисления:

2RCOO − − 2e → R-R + 2CO 2

Суммарное уравнение электролиза:

А) раствора Na 2 CO 3:

2H 2 O → 2H 2 (на катоде) + O 2 (на аноде)

Б) раствора Cu(NO 3) 2:

2Cu(NO 3) 2 + 2H 2 O → 2Cu(на катоде) + 4HNO 3 + O 2 (на аноде)

В) раствора AuCl 3:

2AuCl 3 → 2Au(на катоде) + 3Cl 2 (на аноде)

Г) раствора BaCl 2:

BaCl 2 + 2H 2 O → H 2 (на катоде) + Ba(OH) 2 + Cl 2 (на аноде)

Установите соответствие между названием соли и отношением этой соли к гидролизу.

Ответ: А-2; Б-3; В-2; Г-1

Пояснение:

Гидролиз солей – взаимодействие солей с водой, приводящее к присоединению катиона водорода H + молекулы воды к аниону кислотного остатка и (или) гидроксильной группы OH − молекулы воды к катиону металла. Гидролизу подвергаются соли, образованные катионами, соответствующими слабым основаниям, и анионами, соответствующими слабым кислотам.

А) Стеарат натрия – соль, образованная стеариновой кислотой (слабой одноосновной карбоновой кислотой алифатического ряда) и гидроксидом натрия (щелочью – сильным основанием), следовательно, подвергается гидролизу по аниону.

C 17 H 35 COONa → Na + + C 17 H 35 COO −

C 17 H 35 COO − + H 2 O ↔ C 17 H 35 COOH + OH − (образование слабодиссоциирующей карбоновой кислоты)

Среда раствора щелочная (pH > 7):

C 17 H 35 COONa + H 2 O ↔ C 17 H 35 COOH + NaOH

Б) Фосфат аммония – соль, образованная слабой ортофосфорной кислотой и аммиаком (слабым основанием), следовательно, подвергается гидролизу и по катиону, и по аниону.

(NH 4) 3 PO 4 → 3NH 4 + + PO 4 3-

PO 4 3- + H 2 O ↔ HPO 4 2- + OH − (образование слабодиссоциирующего гидрофосфат-иона)

NH 4 + + H 2 O ↔ NH 3 · H 2 O + H + (образование растворенного в воде аммиака)

Среда раствора близка к нейтральной (pH ~ 7).

В) Сульфид натрия – соль, образованная слабой сероводородной кислотой и гидроксидом натрия (щелочью – сильным основанием), следовательно, подвергается гидролизу по аниону.

Na 2 S → 2Na + + S 2-

S 2- + H 2 O ↔ HS − + OH − (образование слабодиссоциирующего гидросульфид-иона)

Среда раствора щелочная (pH > 7):

Na 2 S + H 2 O ↔ NaHS + NaOH

Г) Сульфат бериллия — соль, образованная сильной серной кислотой и гидроксидом бериллия (слабым основанием), следовательно, подвергается гидролизу по катиону.

BeSO 4 → Be 2+ + SO 4 2-

Be 2+ + H 2 O ↔ Be(OH) + + H + (образование слабодиссоциирующего катиона Be(OH) +)

Среда раствора кислая (pH

2BeSO 4 + 2H 2 O ↔ (BeOH) 2 SO 4 + H 2 SO 4

Установите соответствие между способом воздействия на равновесную систему

MgO (тв.) + CO 2(г) ↔ MgCO 3(тв.) + Q

и смещением химического равновесия в результате этого воздействия

Ответ: А-1; Б-2; В-2; Г-3 Пояснение:

Данная реакция находится в химическом равновесии, т.е. в таком состоянии, когда скорость прямой реакции равна скорости обратной. Смещение равновесия в нужном направлении достигается изменением условий реакции.

Принцип Ле-Шателье: если на равновесную систему воздействовать извне, изменяя какой-нибудь из факторов, определяющих положение равновесия, то в системе усилится то направление процесса, которое ослабляет это воздействие.

Факторы, определяющие положение равновесия:

давление : увеличение давления смещает равновесие в сторону реакции, ведущей к уменьшению объема (наоборот, уменьшение давления смещает равновесие в сторону реакции, ведущей к увеличению объема)

температура : повышение температуры смещает равновесие в сторону эндотермической реакции (наоборот, понижение температуры смещает равновесие в сторону экзотермической реакции)

концентрации исходных веществ и продуктов реакции : увеличение концентрации исходных веществ и удаление продуктов из сферы реакции смещают равновесие в сторону прямой реакции (наоборот, уменьшение концентрации исходных веществ и увеличение продуктов реакции смещают равновесие в сторону обратной реакции)

катализаторы не влияют на смещение равновесия, а только ускоряют его достижение .

Таким образом,

А) поскольку реакция получения карбоната магния является экзотермической, уменьшение температуры будет способствовать смещению равновесия в сторону прямой реакции;

Б) углекислый газ является исходным веществом в получении карбоната магния, следовательно, уменьшение его концентрации приведет к смещению равновесия в сторону исходных веществ, т.к. в сторону обратной реакции;

В) оксид магния и карбонат магния являются твердыми веществами, газом является только CO 2 , поэтому его концентрация будет оказывать влияние на давление в системе. При уменьшении концентрации углекислого газа снижается давление, следовательно, равновесие реакции смещается в сторону исходных веществ (обратная реакция).

Г) введение катализатора не влияет на смещение равновесия.

Установите соответствие между формулой вещества и реагентами, с каждым из которых это вещество может взаимодействовать.

ФОРМУЛА ВЕЩЕСТВА РЕАГЕНТЫ

1) H 2 O, NaOH, HCl

2) Fe, HCl, NaOH

3) HCl, HCHO, H 2 SO 4

4) O 2 , NaOH, HNO 3

5) H 2 O, CO 2 , HCl

Ответ: А-4; Б-4; В-2; Г-3

Пояснение:

А) Сера – простое вещество, способно сгорать в кислороде с образованием диоксида серы:

S + O 2 → SO 2

Сера (как и галогены) в щелочных растворах диспропорционирует, в результате чего образуются сульфиды и сульфиты:

3S + 6NaOH → 2Na 2 S + Na 2 SO 3 + 3H 2 O

Концентрированная азотная кислота окисляет серу до S +6 , восстанавливаясь до диоксида азота:

S + 6HNO 3(конц.) → H 2 SO 4 + 6NO 2 + 2H 2 O

Б) Оксид форфора (III) – кислотный оксид, следовательно, взаимодействует со щелочами с образованием фосфитов:

P 2 O 3 + 4NaOH → 2Na 2 HPO 3 + H 2 O

Кроме того, оксид фосфора (III) окисляется кислородом воздуха и азотной кислотой:

P 2 O 3 + O 2 → P 2 O 5

3P 2 O 3 + 4HNO 3 + 7H 2 O → 6H 3 PO 4 + 4NO

В) Оксид железа (III) – амфотерный оксид, т.к. проявляет как кислотные, так и основные свойства (реагирует с кислотам и щелочами):

Fe 2 O 3 + 6HCl → 2FeCl 3 + 3H 2 O

Fe 2 O 3 + 2NaOH → 2NaFeO 2 + H 2 O (сплавление)

Fe 2 O 3 + 2NaOH + 3H 2 O → 2Na 2 (растворение)

Fe 2 O 3 вступает с железом в реакцию конпропорционирования с образованием оксида железа (II):

Fe 2 O 3 + Fe → 3FeO

Г) Cu(OH) 2 – не растворимое в воде основание, растворяется сильными кислотами, превращаясь в соответствующие соли:

Cu(OH) 2 + 2HCl → CuCl 2 + 2H 2 O

Cu(OH) 2 + H 2 SO 4 → CuSO 4 + 2H 2 O

Cu(OH) 2 окисляет альдегиды до карбоновых кислот (аналогично реакции «серебряного зеркала»):

HCHO + 4Cu(OH) 2 → CO 2 + 2Cu 2 O↓ + 5H 2 O

Установите соответствие между веществами и реагентом, с помощью которого их можно отличить друг от друга.

Ответ: А-3; Б-1; В-3; Г-5

Пояснение:

А) Две растворимые соли CaCl 2 и KCl можно различить с помощью раствора карбоната калия. Хлорид кальция вступает с ним в обменную реакцию, в результате которой в осадок выпадает карбонат кальция:

CaCl 2 + K 2 CO 3 → CaCO 3 ↓ + 2KCl

Б) Растворы сульфита и сульфата натрия можно различить индикатором – фенолфталеином.

Сульфит натрия – соль, образованная слабой неустойчивой сернистой кислотой и гидроксидом натрия (щелочью – сильным основанием), следовательно, подвергается гидролизу по аниону.

Na 2 SO 3 → 2Na + + SO 3 2-

SO 3 2- + H 2 O ↔ HSO 3 — + OH — (образование малодиссоциирующего гидросульфит-иона)

Среда раствора щелочная (pH > 7), окраска индикатора фенолфталеина в щелочной среде малиновая.

Сульфат натрия – соль, образованная сильной серной кислотой и гидроксидом натрия (щелочью – сильным основанием), не гидролизуется. Среда раствора нейтральная (pH = 7), окраска индикатора фенолфталеина в нейтральной среде бледно-розовая.

В) Соли Na 2 SO 4 и ZnSO 4 также можно различить с помощью раствора карбоната калия. В обменную реакцию с карбонатом калия вступает сульфат цинка, в результате которой в осадок выпадает карбонат цинка:

ZnSO 4 + K 2 CO 3 → ZnCO 3 ↓ + K 2 SO 4

Г) Соли FeCl 2 и Zn(NO 3) 2 можно различить раствором нитрата свинца. При его взаимодействии с хлоридом железа образуется малорастворимое вещество PbCl 2:

FeCl 2 + Pb(NO 3) 2 → PbCl 2 ↓+ Fe(NO 3) 2

Установите соответствие между реагирующими веществами и углеродсодержащими продуктами их взаимодействия.

РЕАГИРУЮЩИЕ ВЕЩЕСТВА

А) CH 3 -C≡CH + H 2 (Pt) →

Б) CH 3 -C≡CH + H 2 O (Hg 2+) →

В) CH 3 -C≡CH + KMnO 4 (H +) →

Г) CH 3 -C≡CH + Ag 2 O (NH 3) →

ПРОДУКТ ВЗАИМОДЕЙСТВИЯ

1) CH 3 -CH 2 -CHO

2) CH 3 -CO-CH 3

3) CH 3 -CH 2 -CH 3

4) CH 3 -COOH и CO 2

5) CH 3 -CH 2 -COOAg

6) CH 3 -C≡CAg

Ответ: А-3; Б-2; В-4; Г-6

Пояснение:

А) Пропин присоединяет водород, в его избытке превращаясь в пропан:

CH 3 -C≡CH + 2H 2 → CH 3 -CH 2 -CH 3

Б) Присоединение воды (гидратация) алкинов в присутствии солей двухвалентной ртути, в результате чего образуются карбонильные соединения, является реакцией М.Г. Кучерова. Гидратация пропина приводит к образованию ацетона:

CH 3 -C≡CH + H 2 O → CH 3 -CO-CH 3

В) Окисление пропина перманганатом калия в кислой среде приводит к разрыву тройной связи в алкине, в результате чего образуются уксусная кислота и углекислый газ:

5CH 3 -C≡CH + 8KMnO 4 + 12H 2 SO 4 → 5CH 3 -COOH + 5CO 2 + 8MnSO 4 + 4K 2 SO 4 + 12H 2 O

Г) Пропинид серебра образуется и выпадает в осадок при пропускании пропина через аммиачный раствор оксида серебра. Эта реакция служит для обнаружения алкинов с тройной связью на конце цепи.

2CH 3 -C≡CH + Ag 2 O → 2CH 3 -C≡CAg↓ + H 2 O

Установите соответствие между реагирующими веществами и органическим веществом, которое является продуктом реакции.

ПРОДУКТ ВЗАИМОДЕЙСТВИЯ

5) (CH 3 COO) 2 Cu

Ответ: А-4; Б-6; В-1; Г-6

Пояснение:

А) При окислении этилового спирта оксидом меди (II) образуется ацетальдегид, при этом происходит восстановление оксида до металла:

Б) При действии на спирт концентрированной серной кислоты при температуре выше 140 0 C протекает реакция внутримолекулярной дегидратации – отщепление молекулы воды, что приводит к образованию этилена:

В) Спирты бурно реагируют со щелочными и щелочноземельными металлами. Активный металл замещает водород в гидроксильной группе спирта:

2CH 3 CH 2 OH + 2K → 2CH 3 CH 2 OK + H 2

Г) В спиртовом растворе щелочи спирты подвергаются реакции элиминирования (отщепления). В случае с этанолом образуется этилен:

CH 3 CH 2 Cl + KOH (спирт.) → CH 2 =CH 2 + KCl + H 2 O

Используя метод электронного баланса, составьте уравнение реакции:

В данной реакции хлорноватая кислота является окислителем, поскольку содержащийся в ней хлор понижает степень окисления с +5 до -1 в HCl. Следовательно, восстановителем является кислотный оксид фосфора (III), где фосфор повышает степень окисления с +3 до максимальной +5, превращаясь в ортофосфорную кислоту.

Составим полуреакции окисления и восстановления:

Cl +5 + 6e → Cl −1 |2

2P +3 – 4e → 2P +5 |3

Уравнение окислительно-восстановительной реакции запишем в виде:

3P 2 O 3 + 2HClO 3 + 9H 2 O → 2HCl + 6H 3 PO 4

Медь растворили в концентрированной азотной кислоте. Выделившийся газ пропустили над нагретым порошком цинка. Образовавшееся твердое вещество добавили к раствору гидроксида натрия. Через полученный раствор пропустили избыток углекислого газа, при этом наблюдали образование осадка.
Напишите уравнения четырех описанных реакций.

1) При растворении меди в концентрированной азотной кислоте медь окисляется до Cu +2 , при этом выделяется бурый газ:

Cu + 4HNO 3(конц.) → Cu(NO 3) 2 + 2NO 2 + 2H 2 O

2) При пропускании бурого газа над нагретым порошком цинка цинк окисляется, а диоксид азота восстанавливается до молекулярного азота (предполагаемый многими, со ссылкой на википедию, нитрат цинка при нагревании не образуется, так как термически неустойчив):

4Zn + 2NO 2 → 4ZnO + N 2

3) ZnO – амфотерный оксид, растворяется в растворе щелочи, превращаясь в тетрагидроксоцинкат:

ZnO + 2NaOH + H 2 O → Na 2

4) При пропускании через раствор тетрагидроксоцинката натрия избытка углекислого газа образуется кислая соль – гидрокарбонат натрия, в осадок выпадает гидроксид цинка:

Na 2 + 2CO 2 → Zn(OH) 2 ↓ + 2NaHCO 3

Напишите уравнения реакций, с помощью которых можно осуществить следующие превращения:

При написании уравнений реакций используйте структурные формулы органических веществ.

1) Наиболее характерными для алканов являются реакции свободнорадикального замещения, в ходе которых атом водорода замещается на атом галогена. В реакции бутана с бромом преимущественно происходит замещение атома водорода при вторичном атоме углерода, в результате чего образуется 2-бромбутан. Это связано с тем, радикал с неспаренным электроном при вторичном атоме углерода является более устойчивым по сравнению со свободным радикалом с неспаренным электроном при первичном атоме углерода:

2) При взаимодействии 2-бромбутана со щелочью в спиртовом растворе образуется двойная связь в результате отщепления молекулы бромоводорода (правило Зайцева: при отщеплении галогеноводорода от вторичных и третичных галогеналканов атом водорода отщепляется от наименее гидрированного атома углерода):

3) Взаимодействие бутена-2 с бромной водой или раствором брома в органическом растворителе приводит к быстрому обесцвечиванию этих растворов в результате присоединения молекулы брома к бутену-2 и образования 2,3-дибромбутана:

CH 3 -CH=CH-CH 3 + Br 2 → CH 3 -CHBr-CHBr-CH 3

4) При взаимодействии на дибромпроизводное, в котором атомы галогенов находятся при соседних атомах углерода (или при одном и том же атоме), спиртового раствора щелочи происходит отщепление двух молекул галогеноводорода (дегидрогалогенирование) и образование тройной связи:

5) В присутствии солей двухвалентной ртути алкины присоединяют воду (гидратация) с образованием карбонильных соединений:

Смесь порошков железа и цинка реагирует с 153 мл 10%-ного раствора соляной кислоты (ρ = 1,05 г/мл). На взаимодействие с такой же массой смеси требуется 40 мл 20%-ного раствора гидроксида натрия (ρ = 1,10 г/мл). Определите массовую долю железа в смеси.
В ответе запишите уравнения реакций, которые указаны в условии задачи, и приведите все необходимые вычисления.

Ответ: 46,28%

При сгорании 2,65 г органического вещества получили 4,48 л углекислого газа (н.у.) и 2,25 г воды.

Известно, что при окислении этого вещества сернокислым раствором перманганата калия образуется одноосновная кислота и выделяется углекислый газ.

На основании данных условия задания:

1) произведите вычисления, необходимые для установления молекулярной формулы органического вещества;

2) запишите молекулярную формулу исходного органического вещества;

3) составьте структурную формулу этого вещества, которая однозначно отражает порядок связи атомов в его молекуле;

4) напишите уравнение реакции окисления этого вещества сернокислым раствором перманганата калия.

Ответ:
1) C x H y ; x = 8, y = 10
2) C 8 H 10
3) C 6 H 5 -CH 2 -CH 3 — этилбензол

4) 5C 6 H 5 -CH 2 -CH 3 + 12KMnO 4 + 18H 2 SO 4 → 5C 6 H 5 -COOH + 5CO 2 + 12MnSO 4 + 6K 2 SO 4 + 28H 2 O

Пробные варианты егэ 2021 каждую неделю

пробные варианты егэ 2021 каждую неделю

    Информация Варианты (10) Решения вариантов (8) Курс по подготовке к ЕГЭ 2020 ВКонтакте Instagram Обсуждения (19)

Реши пробный вариант ЕГЭ 2020 от МФТИ и проверь свои знания перед предстоящим экзаменом!

Условия Нового 5 варианта доступны для решения До 1 июля!

Решения и условия 1-4 вариантов ЕГЭ доступны на вкладке «Решения вариантов».

Московский физико-технический институт подготовил для вас серию Пробных вариантов ЕГЭ 2020 по физике и математике, которые каждый желающий может прорешать и тем самым проверить свои знания перед настоящим экзаменом.

Условия Нового 5 варианта доступны для решения До 1 июля!

    Информация Варианты (10) Решения вариантов (8) Курс по подготовке к ЕГЭ 2020 ВКонтакте Instagram Обсуждения (19)

Реши пробный вариант ЕГЭ 2020 от МФТИ и проверь свои знания перед предстоящим экзаменом!

Условия Нового 5 варианта доступны для решения До 1 июля!

Решения и условия 1-4 вариантов ЕГЭ доступны на вкладке «Решения вариантов».

Московский физико-технический институт подготовил для вас серию Пробных вариантов ЕГЭ 2020 по физике и математике, которые каждый желающий может прорешать и тем самым проверить свои знания перед настоящим экзаменом.

Реши пробный вариант ЕГЭ 2020 от МФТИ и проверь свои знания перед предстоящим экзаменом!

Обсуждения 19.

Abitu. net

24.04.2019 19:12:30

2019-04-24 19:12:30

Мы вновь начинаем проект «Пробный ЕГЭ каждую неделю».

Каждую неделю будут публиковатьcя бесплатные пробные варианты ЕГЭ — доступные по хэшу #probny_ege@ege100ballov

В понедельник публикуются сами варианты. Ответы к вариантам — в пятницу.

Подписка на НАШИ ПРОБНЫЕ ВАРИАНТЫ ЕГЭ-2019!
Подписаться прямо сейчас: https://vk. com/app5748831_-10175642
Теперь есть шанс первыми узнавать о вышедших пробных и прорешать, пока вариант не попался на контрольной! Прямо в ЛС!
Рассылка будет осуществляться в понедельник и пятницу.

Пробники публикуются бесплатно, без ограничений для индивидуального и коллективного разбора.
Технические предметы публикуются по нечетным неделям, гуманитарные по четным неделям

Выпуск №1, технические предметы от 03.09.2018
ОТВЕТЫ: https://yadi. sk/d/FQkUsYLYv73kGA

Русский язык https://yadi. sk/i/1RmLAWld3aoNGX
Математика (профиль) https://yadi. sk/i/EvT7nCUl3aoNGN
Математика (база) https://yadi. sk/i/njag9jbG3aoNEh
Химия https://yadi. sk/i/xh-NPFzs3aoNGd
Физика https://yadi. sk/i/1dfrW4gk3aoNHZ
География https://yadi. sk/i/odkqxTF83aoNF3
Информатика https://yadi. sk/i/yFpPjktU3aoNGH
Биология https://yadi. sk/i/3hOXtNXh4aoNEn

СОСТАВИТЕЛИ НАШИХ ВАРИАНТОВ:
Русский язык Елена Хребтищева
Математика (профиль и база) Евгений Пифагор
Химия Иван Ермолаев
География Вова Цой
Информатика Евгений Джобс

Русский язык https://yadi. sk/i/1RmLAWld3aoNGX
Математика (профиль) https://yadi. sk/i/EvT7nCUl3aoNGN
Математика (база) https://yadi. sk/i/njag9jbG3aoNEh
Химия https://yadi. sk/i/xh-NPFzs3aoNGd
Физика https://yadi. sk/i/1dfrW4gk3aoNHZ
География https://yadi. sk/i/odkqxTF83aoNF3
Информатика https://yadi. sk/i/yFpPjktU3aoNGH
Биология https://yadi. sk/i/3hOXtNXh4aoNEn

Мы вновь начинаем проект «Пробный ЕГЭ каждую неделю».

Каждую неделю будут публиковатьcя бесплатные пробные варианты ЕГЭ — доступные по хэшу #probny_ege@ege100ballov

В понедельник публикуются сами варианты. Ответы к вариантам — в пятницу.

Подписка на НАШИ ПРОБНЫЕ ВАРИАНТЫ ЕГЭ-2019!
Подписаться прямо сейчас: https://vk. com/app5748831_-10175642
Теперь есть шанс первыми узнавать о вышедших пробных и прорешать, пока вариант не попался на контрольной! Прямо в ЛС!
Рассылка будет осуществляться в понедельник и пятницу.

Пробники публикуются бесплатно, без ограничений для индивидуального и коллективного разбора.
Технические предметы публикуются по нечетным неделям, гуманитарные по четным неделям

Выпуск №1, технические предметы от 03.09.2018
ОТВЕТЫ: https://yadi. sk/d/FQkUsYLYv73kGA

Русский язык https://yadi. sk/i/1RmLAWld3aoNGX
Математика (профиль) https://yadi. sk/i/EvT7nCUl3aoNGN
Математика (база) https://yadi. sk/i/njag9jbG3aoNEh
Химия https://yadi. sk/i/xh-NPFzs3aoNGd
Физика https://yadi. sk/i/1dfrW4gk3aoNHZ
География https://yadi. sk/i/odkqxTF83aoNF3
Информатика https://yadi. sk/i/yFpPjktU3aoNGH
Биология https://yadi. sk/i/3hOXtNXh4aoNEn

СОСТАВИТЕЛИ НАШИХ ВАРИАНТОВ:
Русский язык Елена Хребтищева
Математика (профиль и база) Евгений Пифагор
Химия Иван Ермолаев
География Вова Цой
Информатика Евгений Джобс

Выпуск №1, технические предметы от 03.09.2018
ОТВЕТЫ: https://yadi. sk/d/FQkUsYLYv73kGA

Выпуск 1, технические предметы от 03.

M. vk. com

23.09.2020 4:43:58

2020-09-23 04:43:58

Единый государственный экзамен по математике 2021 года в 11 классе пройдет в следующие даты:

    базовый и профильный уровни — досрочный период базовый и профильный уровни — резервный день досрочного периода Базовый уровень — отменен! Профильный уровень — основной период профильный уровень — резервный день

Тренировочные варианты ЕГЭ по математике на 2021 год

Пробные варианты ЕГЭ по математике 2021 базового уровня с ответами и решениями от Школы Пифагора:

    Тренировочный вариант № 1 (база) Тренировочный вариант № 2 (база) Тренировочный вариант № 3 (база) Тренировочный вариант № 4 (база) Тренировочный вариант № 5 (база) Тренировочный вариант № 6 (база) Тренировочный вариант № 7 (база) Тренировочный вариант № 8 (база) Тренировочный вариант № 9 (база) Тренировочный вариант № 10 (база) Тренировочный вариант № 11 (база) Тренировочный вариант № 12 (база) Тренировочный вариант № 13 (база) Тренировочный вариант № 14 (база) Тренировочный вариант № 15 (база) Тренировочный вариант № 16 (база) Тренировочный вариант № 17 (база) Тренировочный вариант № 18 (база) Тренировочный вариант № 19 (база) Тренировочный вариант № 20 (база) Тренировочный вариант № 21 (база) Тренировочный вариант № 22 (база) Тренировочный вариант № 23 (база) Тренировочный вариант № 24 (база) Тренировочный вариант № 25 (база) Тренировочный вариант № 26 (база) Тренировочный вариант № 27 (база) Тренировочный вариант № 28 (база) Тренировочный вариант № 29 (база) Тренировочный вариант № 30 (база) Тренировочный вариант № 31 (база) Тренировочный вариант № 32 (база) Тренировочный вариант № 33 (база) Тренировочный вариант № 34 (база) Тренировочный вариант № 35 (база)

Пробные варианты ЕГЭ по математике 2021 профильного уровня с ответами и решениями от Школы Пифагора:

    Тренировочный вариант № 1 (профиль) Тренировочный вариант № 2 (профиль) Тренировочный вариант № 3 (профиль) Тренировочный вариант № 4 (профиль) Тренировочный вариант № 5 (профиль) Тренировочный вариант № 6 (профиль) Тренировочный вариант № 7 (профиль) Тренировочный вариант № 8 (профиль) Тренировочный вариант № 9 (профиль) Тренировочный вариант № 10 (профиль) Тренировочный вариант № 11 (профиль) Тренировочный вариант № 12 (профиль) Тренировочный вариант № 13 (профиль) Тренировочный вариант № 14 (профиль) Тренировочный вариант № 15 (профиль) Тренировочный вариант № 16 (профиль) Тренировочный вариант № 17 (профиль) Тренировочный вариант № 18 (профиль) Тренировочный вариант № 19 (профиль) Тренировочный вариант № 20 (профиль) Тренировочный вариант № 21 (профиль) Тренировочный вариант № 22 (профиль) Тренировочный вариант № 23 (профиль) Тренировочный вариант № 24 (профиль) Тренировочный вариант № 25 (профиль) Тренировочный вариант № 26 (профиль) Тренировочный вариант № 27 (профиль) Тренировочный вариант № 28 (профиль) Тренировочный вариант № 29 (профиль) Тренировочный вариант № 30 (профиль) Тренировочный вариант № 31 (профиль) Тренировочный вариант № 32 (профиль) Тренировочный вариант № 33 (профиль) Тренировочный вариант № 34 (профиль) Тренировочный вариант № 35 (профиль)

Варианты ЕГЭ по математике от Решу ЕГЭ

Тренировочные варианты по математике базового уровня с ответами и решениями от портала Решу ЕГЭ (Сдам ГИА):

Все варианты ЕГЭ по математике представлены в формате pdf — вы легко можете скачать их и распечатать.

Реальные варианты ЕГЭ по математике

Варианты досрочного экзамена официально предоставлены ФИПИ.

    базовый и профильный уровни — досрочный период базовый и профильный уровни — резервный день досрочного периода Базовый уровень — отменен! Профильный уровень — основной период профильный уровень — резервный день

Пробные варианты ЕГЭ по математике 2021 базового уровня с ответами и решениями от Школы Пифагора:

    Тренировочный вариант № 1 (база) Тренировочный вариант № 2 (база) Тренировочный вариант № 3 (база) Тренировочный вариант № 4 (база) Тренировочный вариант № 5 (база) Тренировочный вариант № 6 (база) Тренировочный вариант № 7 (база) Тренировочный вариант № 8 (база) Тренировочный вариант № 9 (база) Тренировочный вариант № 10 (база) Тренировочный вариант № 11 (база) Тренировочный вариант № 12 (база) Тренировочный вариант № 13 (база) Тренировочный вариант № 14 (база) Тренировочный вариант № 15 (база) Тренировочный вариант № 16 (база) Тренировочный вариант № 17 (база) Тренировочный вариант № 18 (база) Тренировочный вариант № 19 (база) Тренировочный вариант № 20 (база) Тренировочный вариант № 21 (база) Тренировочный вариант № 22 (база) Тренировочный вариант № 23 (база) Тренировочный вариант № 24 (база) Тренировочный вариант № 25 (база) Тренировочный вариант № 26 (база) Тренировочный вариант № 27 (база) Тренировочный вариант № 28 (база) Тренировочный вариант № 29 (база) Тренировочный вариант № 30 (база) Тренировочный вариант № 31 (база) Тренировочный вариант № 32 (база) Тренировочный вариант № 33 (база) Тренировочный вариант № 34 (база) Тренировочный вариант № 35 (база)

Пробные варианты ЕГЭ по математике 2021 профильного уровня с ответами и решениями от Школы Пифагора:

    Тренировочный вариант № 1 (профиль) Тренировочный вариант № 2 (профиль) Тренировочный вариант № 3 (профиль) Тренировочный вариант № 4 (профиль) Тренировочный вариант № 5 (профиль) Тренировочный вариант № 6 (профиль) Тренировочный вариант № 7 (профиль) Тренировочный вариант № 8 (профиль) Тренировочный вариант № 9 (профиль) Тренировочный вариант № 10 (профиль) Тренировочный вариант № 11 (профиль) Тренировочный вариант № 12 (профиль) Тренировочный вариант № 13 (профиль) Тренировочный вариант № 14 (профиль) Тренировочный вариант № 15 (профиль) Тренировочный вариант № 16 (профиль) Тренировочный вариант № 17 (профиль) Тренировочный вариант № 18 (профиль) Тренировочный вариант № 19 (профиль) Тренировочный вариант № 20 (профиль) Тренировочный вариант № 21 (профиль) Тренировочный вариант № 22 (профиль) Тренировочный вариант № 23 (профиль) Тренировочный вариант № 24 (профиль) Тренировочный вариант № 25 (профиль) Тренировочный вариант № 26 (профиль) Тренировочный вариант № 27 (профиль) Тренировочный вариант № 28 (профиль) Тренировочный вариант № 29 (профиль) Тренировочный вариант № 30 (профиль) Тренировочный вариант № 31 (профиль) Тренировочный вариант № 32 (профиль) Тренировочный вариант № 33 (профиль) Тренировочный вариант № 34 (профиль) Тренировочный вариант № 35 (профиль)

Тренировочные варианты по математике базового уровня с ответами и решениями от портала Решу ЕГЭ (Сдам ГИА):

Тренировочные варианты ЕГЭ по математике на 2021 год

Тренировочный вариант 16 база.

School-mosreg. ru. com

13.03.2018 14:15:26

2018-03-13 14:15:26

Источники:

Https://rus-ege. sdamgia. ru/

Https://4ege. ru/trening-matematika/

Https://reshu-ege-oge. com/ege_matematika_bazovyj_uroven. html

Https://abitu. net/online_ege

Https://m. vk. com/topic-10175642_39008096

Https://school-mosreg. ru. com/ege/matematika/

ЕГЭ-2022 по русскому языку — Фонова Светлана Анатольевна

Я столько хотела бы сказать о ЕГЭ-Сфере, жаль, что нужные слова теряются в самый неподходящий момент. Я просто хочу сказать огромное спасибо. Спасибо за знания, за веру в меня, за любовь, за поддержку, за приятную атмосферу,за заботу, за мотивацию и, конечно, за подготовку к экзаменам. В этом году я 85% своего времени проводила именно в ЕГЭ-Сфере, 10% дома за заданиями из ЕГЭ-сферы и 5% где-то между школой и домом. Но это того стоило… Я бежала на занятия к Ирине Сергеевне. Ирина Сергеевна — Человек с большой буквы. Спасибо Вам огромное. Вы всегда звонили, писали, поддерживали, помогали и советовали. Это было всегда очень нужно и вовремя, СПАСИБО. К Зое Александровне я бежала чуть менее быстро… но только лишь потому, что это было утро воскресенья — доброе утро воскресенья. Отдельное спасибо Светлане Анатольевне за экспресс-курс по тонкостям ЕГЭ по русскому за 3 занятия. Вчерашний праздник- день семьи, любви и верности- это праздник ЕГЭ-Сферы, ведь это большая семья, в которой царит любовь. А мы — ученики ЕГЭ-Сферы- всегда будем верны этому месту. СПАСИБО БОЛЬШОЕ!!
Сюнякова Вероника. 2019 год .

Цель занятий была сдать ЕГЭ на высокие баллы. Её достигли, потому что я технарь и 72, вроде, это очень хороший для меня результат. Я довольна. Поступила в МИРЭА РТУ. Уроки проходили отлично. Было интересно, особенно нравилось обсуждать аргументы для сочинения. Научилась многому. Информации хватало. Всё нравилось. Большое спасибо преподавателю!
Рябикина Марина

Здравствуйте! Хочу поделиться приятными впечатлениями о работе со специалистом. Светлана Анатольевна за короткий срок смогла систематизировать имеющиеся знания и подтянуть письменную часть экзамена. Занятия проходили в уютной, приятной атмосфере. Она приятная в общении, легко идет на контакт с ребенком. По возникшим вопросам можно было обратиться в любое время. Мы достигли ожидаемого высокого результата. Стоимость вполне соответствует проделанной работе. Я обязательно порекомендую Светлану Анатольевну своим друзьям и знакомым.
Рычалова Инна

Спасибо вам большое за прекрасную подготовку и душевные занятия. Я и не думала , что за маленький срок сумею подготовиться к ЕГЭ. Я довольна своим результатом (конечно допустила пару глупых ошибок, а так могла бы написать на больший балл). Ещё раз спасибо огромное❤
Ярыгина Екатерина

Здравствуйте. Я хочу рассказать про курсы, которые я посещал в прошлом году, и каких результатов я добился. Сразу скажу, что я начал ходить на них только в январе. До этого я думал:» Да ладно, как-нибудь справлюсь». Но потом пришли результаты по пробному тестированию, и они были ужасными(( Я сразу же записался на эти курсы ,и благодаря преподавателям, я смог достичь отличных результатов и поступить в престижный вуз. Учителя объясняли мне доступным языком и я советую всем записаться именно сюда.
Герцог Евгений

Как изучать общую химию | по Med School Motivation

«Я не могу этого сделать. Я просто не могу этого сделать. Не каждый достаточно умен, чтобы получить пятёрку по химии ».

Сколько раз вы слышали вариант из вышеперечисленного? Вы можете заменить химию любым предметом или занятием, которым вы пытаетесь овладеть, и вы обнаружите то же самое: многие люди ограничивают себя в том, чего они могут достичь.

В академических кругах есть интеллектуальный барьер. Худшая теория, которая когда-либо исходила из психологии, заключается в том, что каждый наш мозг настроен на выполнение определенной задачи.Некоторые из нас одарены математикой, искусством, языками. Мы все слышали, как обучаются левое и правое полушарие. Мы представляем, что каждый из нас был задуман и рожден не с чистого листа, а со списком качеств, которые в конечном итоге определят, станем ли мы химиками или модельерами.

Ну, это не так.

Ваш мозг — это эластичный элемент оборудования. Все, что вы хотите научиться делать, какую бы задачу вы ни хотели выполнить, вы можете. Просто нужно уметь это делать.

Я проповедую на этом веб-сайте усердную работу, потому что это ключевой ингредиент в достижении любых целей . Но в академическом мире, особенно если вы пытаетесь изучить сложный предмет, недостаточно просто много работать.

Тоже надо работать в правильном направлении .

Когда я был моложе, я думал, что для меня невозможно выучить иностранный язык. Я думал, что по какой-то причине мой мозг не был устроен таким образом, чтобы позволить моему разуму думать на другом языке.После семи лет изучения испанского в старшей школе и колледжа я не мог поддерживать элементарный разговор, не говоря уже о том, чтобы что-либо писать. Однако причина того, что у меня был такой небольшой успех, заключалась в том, что я, , учился неправильно.

Мало изучить предмет. Вы не можете просто подходить к каждому курсу одинаково, вы должны сначала сосредоточиться на аспектах, которые принесут вам наибольшую пользу, а затем найти способ сохранить эти знания. Что касается меня, я изучал испанский, как урок истории.Я постоянно читал учебник и делал сотни бесполезных карточек. Я мог бы рассказать вам, что означает слово на английском языке, но я никогда не смог бы связать их в одно предложение.

С годами я обнаружил, что химия для многих кажется тем же, чем испанский для меня. Если вы когда-нибудь чувствовали разочарование, изучая химию, потерянное, сбитое с толку, ошеломленное, если вы когда-либо чувствовали, что ваши часы учебы потрачены впустую, то примите во внимание тот факт, что не ваш интеллект виноват , а что вы учились неправильно.Вы двигались не в том направлении. Этот пост направит вас на правильный путь.

Я не претендую на звание химика. До колледжа я прослушал только один курс химии в старшем классе старшей школы, и это был базовый курс. Изначально я получил степень бакалавра по английскому языку и творческому письму с несколькими курсами биологии. Когда я вернулся в школу, чтобы получить вторую степень по биохимии, мне пришлось изучать общую и органическую химию. Несмотря на минимальное научное образование, я смог получить 4.0 на моих курсах, в том числе наивысшие баллы по нескольким тестам и отличные оценки по обоим выпускным экзаменам по общей химии. Мне удалось добиться успеха, пока я видел, как многие из моих главных друзей-химиков с многолетним опытом работы в области науки AP получают только средние оценки.

Это потому, что я гений? Отнюдь не. Я не умнее любого другого студента. Я полностью связываю свой успех с учебными привычками, которые я использовал для подготовки к экзаменам, а не с какой-либо склонностью к естественным наукам.

Каждый может научиться добиваться успеха. Независимо от того, какие проблемы у вас были с естествознанием или математикой в ​​прошлом, если вы боретесь с химией, обратите пристальное внимание на свои учебные привычки и то, как вы учитесь, и посмотрите, где вы можете что-то изменить.

Учебник, безусловно, будет вашим самым большим активом в изучении и изучении общей химии. Может быть, это кажется очевидным, но если вы не готовы читать и прорабатывать каждую главу, у вас ничего не получится. Химия — это не английский или история, вы не можете пролистать текст и стать профессиональным.Вы должны изучать материал, изучать его по частям и повторять, пока вы полностью не поймете его.

Прочтите каждую главу в учебнике ПЕРЕД сопроводительной лекцией по этой главе. Большинство профессоров строят свои учебные программы таким образом, чтобы у них было время для этого. К сожалению, большинство учеников полностью игнорируют чтение до выходных или намного позже того, как оно будет рассмотрено в классе.

Не делайте этого.

Прочтите каждый раздел и, по крайней мере, ознакомьтесь с материалом, который вы будете обсуждать на лекции.Когда я впервые брался за новую главу, я всегда начинал с чтения заголовков разделов и ключевых понятий или выделенных жирным шрифтом пунктов на протяжении всей главы. Это даст вам раннее представление о том, что вы будете изучать и какие концепции появятся в будущем. На прочтение заголовков разделов у вас не должно уйти больше пары минут. После этого приступайте к чтению главы. Подходите к каждому разделу как к отдельной единице. Не переходите к следующему, пока не почувствуете себя комфортно с материалом.

Большая часть чтения учебника химии не имеет для вас значения.Каждый раз, когда вы видите даты, историческую информацию или биографические данные известных химиков, вы можете не обращать на них внимания (если это вас не интересует). Большинство профессоров не собираются ставить вам оценки за этот лишний материал. Как правило, вы будете проходить тестирование по концептуальной работе, которая выделяется среди остальных и сопровождается примерами задач, пошаговыми решениями и дополнительными типовыми вопросами. Сосредоточьтесь на этом материале. Сосредоточьтесь на том, чтобы узнать, как именно решать каждую проблему, и на подсказках, которые помогут вам понять, что вам задает вопрос.

Допустим, во вторник у меня будет лекция по химии, в которой будет рассмотрен материал главы 2, разделы 1–4. Я буду структурировать свое обучение следующим образом:

Понедельник:

  1. Прочтите названия разделов и просмотрите главу, чтобы узнать о ключевых концепциях
  2. Прочтите разделы 1–4, сделав заметки о концептуальной информации
  3. Решите все задачи и образцы вопросов в разделах (включая те, на которые книга может не дать ответов).

Вторник:

  1. Посещайте лекцию , делайте заметки
  2. Просмотрите разделы, которые вызывали у меня наибольшие проблемы, сосредоточьтесь на том, что нужно уточнить
  3. Дополнительные задачи для разделов 1–4, найденные в конце главы

Среда:

  1. Повторите шаги для материала лекции по четвергам

Возьмите за привычку повторять то, что вы узнали, через день или два лекции. Вы хотите, чтобы материал был свежим в вашей памяти.Ничего страшного, если у вас возникли проблемы с учебником в первый раз, когда вы столкнулись с ним; в этом важность посещения лекции, у вас есть возможность повторить то, что вы знаете, и, надеюсь, ваш профессор сможет заполнить пробелы в вашем понимании. В любом случае вы хотите создать несколько слоев усиления материала, чтобы ваш мозг действительно учил и сохранял информацию. Если вы можете структурировать свое обучение таким образом, чтобы постоянно анализировать ранее изученный материал, вам не придется заново учить его, когда придет время сдавать экзамен.

Изучите материал перед лекцией -> посетите лекцию, чтобы закрепить знания -> просмотрите любой материал, с которым у вас все еще могут возникнуть трудности

Хотя этот раздел идет рука об руку с учебником, если есть один совет вы должны сделать вывод из этой статьи, это важность решения практических задач. Общая химия — это решение отдельных групп проблем. Есть общие концепции, которые вам нужно изучить, которые помогут вам собрать воедино все, от поляризации до энтальпии, но это набор задач, на которых вы будете проверяться.

Не относитесь к химии как к обычному научному курсу. В биологии важны чтение и запоминание маршрутов, вам нужно уметь повторять ответы и факты, чтобы успешно сдать тесты. Общая химия похожа на урок математики. Это проблема задает вопрос. Вы будете проверены на вашу способность распознать, о чем спрашивается от вас, какие формулы вам нужно использовать, а затем как решить проблему в правильных единицах и значащих цифрах.

Рабочие проблемы.Все они.

Все учебники химии, которые я просматривал, имеют одинаковую структуру. У вас есть концептуальный раздел, в котором книга исследует , почему стоит за тем, что вы изучаете. Затем у вас есть часть, в которой книга расскажет, как решать связанные проблемы, с пошаговыми инструкциями, за которыми следует ряд примеров задач.

Вот где многие студенты ошибаются.

Вы только что видели, как проблема должна решаться. НЕ пропускайте примеры задач.Не думайте только потому, что вы однажды видели, как проблема решалась, вы ее решили. Вы должны попытаться решить подобную проблему самостоятельно, без руководства. Сделайте это, не глядя на ответ или на способы решения проблемы. Решайте каждый вопрос вручную. Записывая соответствующие детали, которые вам задают, вы начинаете развивать навык распознавания вопросов. Я обнаружил, что химия — это не только решение проблемы, но и понимание того, что именно задает вопрос.Чем больше проблем вы будете решать, тем более привычным станет каждый сценарий.

Вот где проявляется усердие. Нет замены действительному выполнению работы. Лучший способ подойти к учебе — это просто проработать каждую проблему, которую вы видите, читая главы. Не пропускайте ни одного, даже если вы чувствуете, что у вас есть все необходимое. Вы хотите повторения. Вы хотите закрепить свое обучение. Это причина номер один, почему так много студентов говорят, что знают материал дома, но задыхаются от экзаменов.Вы хотите, чтобы рабочие задачи по химии стали второй натурой. Вы хотите проработать столько задач, чтобы оценочные экзамены не отличались от практики, которую вы выполняете дома.

Я настоятельно рекомендую купить рабочие тетради, которые прилагаются к большинству учебников химии. Они не только предлагают дополнительные проблемы для каждой секции, которую нужно проработать, но и предлагают комплексные решения. Не смотрите на решение, пока не поймете, что нужно делать. Легко привыкнуть быстро взглянуть на ответ, чтобы убедиться, что вы правы.Не давайте себе этого преимущества. На экзамене у вас не будет руководства для ответов; так что заставляйте себя работать над каждой проблемой, как над решением головоломки.

Как я уже говорил, лекции нужно использовать для закрепления знаний, которые вы делаете самостоятельно. Лекция — не время для изучения материала. В стандартном курсе химии слишком много всего, чтобы учитель мог понять все во время урока. Лекции предназначены для того, чтобы ваш профессор рассмотрел ключевые концепции химии, поработал над несколькими примерами задач и ответил на несколько вопросов.Вот и все. Если вы поступите в более крупный университет, ваш лекционный зал, вероятно, будет заполнен двумя сотнями других студентов. Если у вас нет возможности получить ответы на свои вопросы, не переживайте, есть часы работы и электронная почта. В большинстве школ также есть учебный центр со специальными репетиторами по химии, если с вашим профессором сложно связаться.

Я хочу повторить тот факт, что вы не должны полагаться на лекционный зал при изучении материала. Лекция — безусловно, самый бесполезный аспект вашего изучения химии.По-прежнему посещайте все занятия, так как то, что вы пройдете, только укрепит ваше обучение и даст вам представление о концепциях, которые, по мнению вашего профессора, важны; и поэтому, вероятно, будут включены в экзамен.

У меня был профессор химии, который никогда не решал ни одной задачи во время занятий. Он читал лекции о сложных концепциях химии на уровне выпускников, сосредоточивал внимание на незначительных деталях и жизни известных химиков и на всем ненужном материале, на котором мы никогда не будем проверяться.Он был одним из ведущих мировых экспертов в области фотохимии и дошел до того, что провел целую лекцию, показывая документальный фильм о нем и его команде, а также об экспериментах, которые они проводили над тем, почему пиво становится тусклым при воздействии света (серьезно). В целом он был блестящим парнем, но явно не интересовался преподаванием химии на уровне бакалавриата. Когда дело доходило до экзаменов, класс стонал, надувался и жаловался, что ничего, что мы узнали на лекции, не было на экзамене. Но это упускает суть.

Вы не научитесь всему из своего лекционного зала. Велика вероятность, что к экзамену вы вообще не узнаете ничего полезного. Не вините систему и не зацикливайтесь на жалобах на ужасного профессора. Колледж — это не только научиться учиться самостоятельно (будучи самоучкой), но и следовать указаниям профессора. Используйте лекционный зал, чтобы закрепить свои знания.

Может быть, у вас есть отличный преподаватель, который просматривает весь материал, разрабатывает примеры в классе и опаздывает, чтобы ответить на все ваши вопросы.Это фантастика, но вы все равно должны взять процесс обучения в свои руки. Несмотря ни на что, всегда старайтесь просмотреть материал перед лекцией. Изучение и сохранение информации — это повторяющийся процесс; повторение работ. Чем больше раз вы встретите материал, тем лучше. Прочтите его один раз, послушайте один раз, еще раз просмотрите.

Есть три отличных способа получить помощь по любому материалу, с которым вы все еще боретесь. Первый — это часы работы вашего профессора. У каждого профессора будет определенное время в течение недели, когда они будут встречаться со студентами и помогать им с вопросами.Используйте этот ресурс. Хотя некоторые профессора, как известно, завалены студентами, ищущими помощи, на протяжении моей карьеры в колледже многие учителя рассказывали, как мало студентов приходило каждую неделю. У вас нет оправдания, чтобы не пойти к учителю, если вам в чем-то понадобится помощь. Их работа не только в том, чтобы учить вас, но и в том, что хороший профессор наставит вас таким образом, чтобы это соответствовало их образу мышления, а это значит, что вы узнаете точный способ ответить на проблему в вопросе , который они создали.Однажды у меня был профессор математического анализа, который задавал невероятно сложные вопросы на экзамене, которые никоим образом не напоминали то, что мы делали в учебнике или домашних заданиях. Я начал посещать его рабочие часы и быстро изучил его мыслительный процесс, что стало большим подспорьем в решении вопросов, с которыми я столкнулся позже на экзамене.

Часто ваш преподаватель не только сможет помочь вам с концепциями, с которыми у вас возникают трудности, но и подскажет, как лучше всего добиться успеха в своем классе; Это переменная, которая может сильно различаться среди профессоров даже на одном и том же факультете.Некоторые профессора труднее других, некоторые ценят определенные навыки или считают определенные темы более важными, чем их коллеги. Не обязательно узнавать своего профессора поближе, но это даст вам некоторое представление, которое только поможет вам сконцентрировать свое внимание на подготовке к экзаменам.

Использование рабочего времени — отличный способ наладить отношения с профессором. На двухстах студенческих лекциях по естествознанию вы просто еще одно лицо. Но если вы идете в рабочее время — с законными вопросами — вы даете учителю знать о себе как о человеке, которому небезразлично изучать материал; удивительно необычная цель среди большинства студентов колледжей.Помните, что большинство профессоров начали преподавать, потому что хотят помочь студентам добиться успеха. Когда-то они были на твоем месте. Они знают, каково быть студентом колледжа и испытывать трудности с предметом. Если вы заботитесь о том, что делаете, и демонстрируете хорошую трудовую этику, большинство профессоров, естественно, будут склонны взять вас под свое крыло и помочь вам.

При этом некоторые профессора не приглашают. Если у вас есть профессор, который не помогает во время лекции, попробуйте его в рабочее время.Но если вы чувствуете, что это пустая трата времени, или если профессор слишком занят или, похоже, не заинтересован в помощи, тогда поищите в другом месте. У меня была напарница по лаборатории, у которой профессор раскладывал пасьянс на своем компьютере все время, пока она была в его офисе. Ясно, что это не тот человек, которому вы хотите помочь.

В каждом кампусе должен быть учебный центр для студентов. В каждом учебном центре будут наставники, студенты-химики, ассистенты и студенты, которые смогут помочь вам и ответить на вопросы.Эти студенты часто работают как профессорские часы, либо по предварительной записи, либо в обычное время, на которое вы можете пойти. Преимущество обучения у сокурсника заключается в том, что он совсем недавно оказался на вашем месте и часто более отчетливо помнит подводные камни изучения общей химии, которые забыл ваш доктор наук с двадцатилетним стажем. Если ваш преподаватель лекционного зала не помогает, я определенно рекомендую обратиться за помощью в услуги учебного центра.

Последняя доступная помощь — это Интернет.Если просто погуглить свой вопрос или концепцию, откроется обширный список веб-сайтов, ответов Yahoo и форумов, на которых студенты получат ответы. Это может быть отличным ресурсом, особенно если вы можете найти ответ, который четко объясняет вам проблему и решает ее шаг за шагом. Но я бы рекомендовал использовать его не так часто. Легко выработать привычку искать в Интернете проблемы, с которыми вы боретесь, вместо того, чтобы тратить время на попытки решить их самостоятельно.

Пользуйтесь Интернетом экономно.Вы должны попытаться решить каждую проблему как можно дальше, прежде чем обращаться за помощью. Бороться — это нормально, тогда вы начинаете устанавливать связи, которые помогают вам расти, но руководствуйтесь здравым смыслом. Если вы потратили двадцать минут на решение одной проблемы, вам помогут. Не двигайся дальше. Правильное распоряжение своим временем имеет решающее значение для учебы. Потратьте слишком много времени на решение одной проблемы, и вы ничего не сделаете или сгорите. Ни то, ни другое не поможет вам в долгосрочной перспективе.

Независимо от того, какой ресурс вы используете, нет ничего постыдного в том, чтобы попросить о помощи.Когда вы получаете свою последнюю оценку в конце семестра, вы не хотите оглядываться назад с сожалением о том, что не получили необходимую помощь.

Скажу честно, я не фанат кружков. Каждая учебная группа, в которой я участвовал, была пустой тратой времени. Забавно снимать отдельную комнату с друзьями в библиотеке и бездельничать, это не весело, сидя на экзамене и проваливая его. Учебные группы неизбежно тратят впустую драгоценное время, которое можно было бы использовать для учебы. Вы будете сбиты с толку, не по теме, вы можете сидеть и просматривать материал, с которым вам не нужна никакая помощь, вам, возможно, придется изменить свое расписание и тратить время на встречи с группой и ожидание, когда все придут.Вы также попадаете в ловушку чрезмерной самоуверенности. Вы и ваши товарищи по учебе можете лишь бегло просмотреть материал, просмотреть простые концепции или часть того, на чем вы будете проходить тестирование; а не подробный обзор всего материала. Не думайте, что вашей учебной группы раз в неделю будет достаточно, чтобы выучить химию.

Если вы собираетесь работать отдельно от учебной группы, используйте это в качестве учебного опыта. Преподавание — один из лучших способов закрепить усвоенный материал.Если вы можете четко и лаконично преподать концепцию другому человеку, значит, вы усвоили ее. Преподавание в группах по изучению — это быстрый способ понять, что вы знаете, а чего нет, и это гораздо более полезно, чем просто слушать, как ваши сверстники спотыкаются о концепциях.

Но, как правило, я избегаю учебных групп любой ценой. Я предпочитаю эффективно распоряжаться своим учебным временем. Мы все согласны с тем, что предпочли бы заниматься другими делами, а не учиться. Не тратьте впустую больше времени, чем нужно.Выполняйте работу и общайтесь позже.

Хотя изучение материала любого курса важно, в конце концов, действительно важно то, насколько хорошо вы сдадите экзамены. Вот пошаговый метод, который я использовал для подготовки к экзаменам, который помог мне набрать наивысшие баллы во всех моих тестах по химии:

  1. Изучите учебное пособие, если его предоставляет ваш профессор. Это отличный способ получить быстрый обзор материала.
  2. Попробуйте пройти пробный экзамен, если его дает ваш профессор.Не помогайте себе с пробным тестом; это отличный показатель того, насколько хорошо вы сдадите экзамен без дополнительного обучения. *
  3. Составьте мысленный список концепций, с которыми вы боролись или ошиблись в учебном пособии / образце теста. Это те части экзамена, на которых вам нужно сосредоточиться больше всего.
  4. Просмотрите все главы учебника для своего теста. ** Делайте это по частям, работая с каждым примером проблемы и типовым вопросом, с которыми вы сталкиваетесь (даже если вы уже работали с этими проблемами раньше).В конце каждой главы проработайте все примеры задач, включая любые дополнительные задачи в конце книги или в прилагаемой рабочей тетради. По сути, решает столько проблем, сколько может . Просмотрите материалы по учебнику, чтобы освежиться, ЕСЛИ это не та концепция, с которой вы столкнулись в образце теста. Уделите больше времени этим разделам и переработайте проблемы, пока не усвоите их.
  5. После того, как вы закончите просмотр глав, пройдите практический тест еще раз. Если вам повезло, что у вас есть профессор, который выкладывает несколько практических тестов в Интернете, то сделайте другой тест, чем раньше.В любом случае, второй раз вы должны значительно улучшить свои показатели.
  6. Если есть что-то, с чем вы все еще боретесь, просмотрите эти разделы книги и исправьте все проблемы еще раз (должно быть в третий раз).
  7. Сдавайте практические экзамены еще раз не реже одного раза в день перед экзаменом. За день до экзамена проработайте их еще раз как минимум дважды.

Вам следует приступить к выполнению следующих шагов за 5–7 дней до теста. Вы не хотите, чтобы вас торопили, но вы также не хотите начинать рецензирование так рано, чтобы весь материал не был свежим в вашей памяти во время экзамена.Большинство курсов построены таким образом, что вы все равно будете изучать новый материал вплоть до лекции перед экзаменом, поэтому примите это во внимание. Я стараюсь, чтобы все мои проблемы были решены, а изучение учебника было выполнено по крайней мере за два дня до экзамена, чтобы я мог провести последний день, просто работая над практическими тестами и просматривая любые последние вопросы, с которыми я борюсь. В день экзамена я ничего не изучаю. Вы можете узнать почему и другие советы на экзаменах в моей статье 5 советов по преодолению тестовой тревожности, не связанной с изучением .

Не буду приукрашивать. Вам придется много работать, если вы хотите получить пятёрку за свой тест. Может показаться утомительным повторение всего материала дважды и в третий раз для финала, но если вы хотите получить пятерку за курс, ваша погрешность невелика. Те случайные вопросы, с которыми вы сталкиваетесь, быстро складываются, поэтому не считайте само собой разумеющимся, что в тесте не будет задана концепция или будет только один вопрос. Вы должны быть знакомы со ВСЕМИ материалами курса.

Некоторые люди учатся быстрее, чем другие, некоторым может быть легче понять концепции. Это может занять у вас меньше пяти дней или больше, но соответственно скорректируйте, сколько времени вы можете посвятить. Если вы готовитесь к заключительному экзамену, не забудьте учесть время, которое вам понадобится для подготовки к другим предметам.

В качестве дополнения, если вы плохо сдаетесь на первом экзамене, не впадайте в режим нервного возбуждения, особенно если ваш профессор позволяет вам сбросить самый низкий результат на экзамене (частая практика вместо кривой).Если у вас нет спада в оценках, но есть кривая, то утешитесь тем фактом, что если вы честно старались изо всех сил и усердно учились, то большинство ваших учеников, вероятно, тоже плохо справились.

Ничего страшного.

После тяжелого экзамена не время жалеть себя. Не время быть несчастным и думать о том, чтобы все бросить. Пришло время проявить мотивацию. Первый экзамен для каждого курса — это опыт обучения, вы точно выясняете, чего ваш профессор ожидает от вас и как он структурирует свой экзамен.Вы узнаете, что вам нужно изучать в следующий раз, а что можно пропустить. Может быть, ваш профессор никогда не будет задавать вам вопросы, связанные с известными химиками, или ваш учитель частично доверяет, или строго относится к сиг-инжирам. Используйте это при подготовке к следующему экзамену.

* У меня не было профессоров, которые раздавали учебные пособия, но у всех были образцы экзаменов на Blackboard / Sakai / вашем портале онлайн-курсов. Если вы все еще не можете найти его, потратьте дополнительное время на решение задач, связанных с книгой, и поищите учебное пособие в Интернете.Хотя вы потеряете преимущество того, что ваш профессор создает вопросы (и, следовательно, будут напоминать экзаменационные вопросы), учебное пособие или образец теста для работы лучше, чем вообще ничего.

** Если это совокупный тест или заключительный экзамен, скорректируйте свои учебные усилия соответствующим образом. Большинство кумулятивных тестов в большей степени опираются на новый охватываемый материал, и только пара вопросов из предыдущих тестов, так что сосредоточьтесь на этом. Некоторые профессора даже указывают, какая часть теста будет охватывать каждую главу.Не тратьте все свое время на то, что будет лишь малой частью вашей оценки. На выпускном экзамене важно просмотреть весь материал, но уделять больше времени тому, с чем вы боретесь. Очевидно, у вас не будет времени переделывать весь учебник, но если вы усердно учились в течение всего семестра, то большая часть материала вернется к вам.

Я не могу достаточно повторить этот момент. Решайте проблемы. Пожалуйста, решите проблемы. Работайте над каждой проблемой, которую можете решить.Проработайте задачи из учебника, рабочую тетрадь, найдите задачи в Интернете, возьмите дополнительную книгу из библиотеки. Чем больше проблем вы решите, тем лучше вы справитесь. Руки опущены, никаких оправданий. Вы должны работать над проблемами, вы должны решать их самостоятельно и не упускать их до тех пор, пока не будете последовательно правильно отвечать на вопросы. Работа над проблемой способствует знакомству. Многократное использование одних и тех же концепций развивает мастерство. Для успешной сдачи экзаменов вам понадобятся и то, и другое. Вы хотите быть в состоянии сразу понять, в чем заключается проблема на экзамене, какие концепции вам нужно будет извлечь, а затем вам понадобится мастерство, чтобы правильно на них ответить.Вы хотите чувствовать себя комфортно на экзамене. Не угадай. Не оставляйте ничего на волю случая.

Все мы слышали поговорку, что удача — это возможность встречает подготовку. Экзамен — это ваша возможность. Ваше обучение — это подготовка. Нам не нужна удача; это просто побочный продукт.

Сначала может показаться невозможным хорошо сдать экзамен по химии; тем более, чтобы получить пятёрку или высший балл. Но это не так. Вы должны следовать системе, чтобы добиться успеха. Ваш профессор не пытается намеренно подвести вас или расстроить.Но они должны усложнить задачу. Химия, особенно общая химия, открывает путь к гораздо более сложным наукам. Это барьер для входа для всех желающих доказать свое здоровье, будущих стоматологов и врачей. Именно здесь начинают свое обучение все химики и фармацевты по полимерам с докторской степенью. Это большое дело. Если вы сможете преуспеть и преуспеть в общей химии, вы не только овладеете базовыми концепциями для своей области, но и научитесь , как выучить , как учиться для каждого класса, с которым вы столкнетесь на этом пути.

Для тех из вас, кто идет в медицинский институт, это бесценный навык. Я уже говорил ранее, что оценки — не единственное, что важно для того, чтобы стать хорошим врачом; но то, что они означают, есть. Курсы в колледже представляют проблему, а успешное их прохождение демонстрирует способность решать эту проблему. Это требует тяжелой работы, изобретательности и выдержки — качества, которые делают , делают хорошего врача. Чтобы поступить в медицинский институт, нужно получить хорошие оценки. Вы должны научиться, чтобы стать хорошим диагностом.

  1. Изучите учебник до до лекции.
  2. Посещайте лекцию, делайте заметки и задавайте вопросы по сложному материалу.
  3. Просмотрите материалов в течение дня лекции. Работа , доп. задач.
  4. При необходимости обратитесь за помощью .
  5. Избегайте учебных групп. Если возможно, научите других.
  6. Просмотрите весь пройденный материал перед экзаменом. Проработайте как можно больше задач и переделайте практические тесты.Начните учиться по крайней мере 5 дней до экзамена.
  7. РАБОТА С ПРОБЛЕМАМИ .

Дополнительные справочные материалы по подготовительным и курсовым работам можно найти на сайте Writer M.D. — Building Better Doctors . Для получения подробных рекомендаций по выставлению оценок в верхнем процентиле MCAT посетите mcatcoach.me

Периодическое издание | Размышления читателя экзамена по химии

В июне мне посчастливилось пройти второй экзамен по химии в Солт-Лейк-Сити, штат Юта.Это был чрезвычайно ценный опыт — познакомиться с тем, как студенты из разных стран отвечают на эти сложные вопросы по химии.

Как правило, читателей по химии обучают и поручают оценивать ответы учащихся на один из семи вопросов с бесплатными ответами, которые задаются на экзамене в этом году. Иногда читатели будут переназначены и переобучены по другому вопросу позже на неделе, в зависимости от того, сколько времени займет оценка вопроса.

© ️ successphoto / Bigstock.ком

К концу недели все читатели прочитали тысячи ответов только на 1 или 2 вопроса, встречая все, от коротких и приятных, конкретных, логичных ответов до сбивающих с толку, излишних и воображаемых неправильных ответов. Прочитав тысячи ответов в ходе двух чтений AP Chemistry, я хотел бы поделиться некоторыми советами, которые учителя AP Chemistry должны рассмотреть со своими учениками во время курса химии AP, чтобы лучше подготовить их к успешной сдаче экзамена.

Ознакомьте студентов с форматом

Исходя из моего опыта, очевидно, что многие студенты идут на экзамен, по сути, слепо понимая, что от них будут спрашивать. Важно, чтобы они понимали формат экзамена, чтобы учесть правильные ожидания и наиболее эффективно спланировать свое время. Вот некоторые детали, которые могут помочь им понять, чего ожидать.

  • Экзамен AP Chemistry состоит из 60 вопросов с несколькими вариантами ответов, 10 из которых относятся к «полевому тестированию».«Вопросы полевого тестирования не будут учитываться при подсчете выпускных оценок учащихся, но у них не будет возможности узнать, какие вопросы какие.
  • Вопросы связаны в книгу со справочной таблицей, и учащимся придется переключаться между вопросами и справочной таблицей.
  • Студенты могут писать заметки и делать «скетчи» в любом месте экзамена, но они обязаны ответить на вопросы из предоставленного листа для ответов в течение 90 минут. Читатели экзамена никогда не видят этих рукописных заметок.
  • Учащимся обычно дается небольшой перерыв между разделами с множественным выбором и свободным ответом (продолжительность этого перерыва зависит от школы). После перерыва студенты получают совершенно новые экзаменационные пакеты и переплетенные книги со свежими справочными таблицами.
  • Раздел бесплатных ответов состоит из трех длинных вопросов, каждый из которых занимает в среднем 23 минуты и дает до десяти баллов за каждый.
  • Есть также четыре коротких вопроса, которые занимают в среднем девять минут и стоят до четырех баллов каждый.
  • Каждый вопрос с бесплатными ответами печатается на одной-двух страницах, без места для работы (мы вернемся к этому позже). После вопроса для ответов в переплетенной книге предоставляется несколько страниц бумаги.

Студентам важно заранее понять формат экзамена, чтобы они могли спланировать, как организовать свое время и работу. Я рекомендую дать студентам практический экзамен, чтобы они имели опыт настройки. Вы можете сдать старый экзамен; но имейте в виду, что у студентов есть доступ к ответам в Интернете.Если вы прошли аудит AP, у вас есть доступ к нескольким практическим экзаменам. Эти практические экзамены безопасны, это означает, что они не должны публиковаться на каких-либо веб-сайтах или доступны учащимся дома. Поэтому учителям AP Chemistry важно, чтобы тесты проводились только в классе, чтобы другие учителя могли использовать их в качестве подлинного инструмента оценки.

Мои студенты сдали все практические экзамены в какой-то момент учебного года, и когда я спросил их, что они считают наиболее ценным для проверки, большинство студентов упомянули эти практические экзамены.Используя эти экзамены, студенты могли практиковать управление временем и фактически видеть формат экзамена до дня экзамена.

Правильная организация бесплатных ответов

Читатели должны прочитать все страницы работы, посвященной оцениваемому вопросу, и следовать ему. Поэтому, если ученик напишет одни ответы рядом с вопросами, а другие на дополнительных страницах, мы оцениваем все это. Следовательно, могут возникнуть серьезные проблемы, когда студенты не знакомы с форматом экзамена и с тем, что от них ожидается.

Как упоминалось ранее, вопросы с бесплатными ответами печатаются с очень ограниченным пространством для учащихся для выработки ответов в отведенном для этого месте непосредственно под вопросом в экзаменационной брошюре. Вместо этого они должны показывать свою работу и ответы в отведенных местах после каждого вопроса в буклете для ответов.

Я понимаю, что студентам может быть сложно постоянно перелистывать страницы вперед и назад, чтобы увидеть вопрос, а затем написать свои ответы.Но это определенно лучше, чем пытаться писать в крошечных промежутках между вопросами. Слишком часто ученики пишут настолько мелко, что их ответ почти невозможно прочитать. Более того, ученики, которые пишут карандашом, также сталкиваются с проблемой письма настолько мягко, что их ответы слишком легкие для чтения. Когда ответ невозможно прочитать, читатели обычно обращаются к своему руководителю таблицы, чтобы помочь им интерпретировать его, и, если ответ по-прежнему остается неразборчивым, он не получает баллов.

Таким образом, вместо того, чтобы писать свои ответы мелким шрифтом, студенты должны писать свои работы и ответы в соответствующих областях экзамена после вопроса.Для длинных письменных ответов места между подвопросами совершенно недостаточно. А за длинные математические вопросы ученики, пытающиеся втиснуть свою работу в это маленькое пространство, потеряют баллы, если читатель не сможет найти ключевые числа и логическую работу, которая помечена и понятна.

Если учащиеся хотят быть уверенными, что получат максимальное количество баллов, они должны записать все ответы в одном месте и записать свои ответы только один раз! Многие студенты пишут свои правильные ответы рядом с вопросом, понимают, что у них есть лишняя бумага, и затем решают передать свою работу.Проблема в том, что большая часть этих студентов неправильно переводит ответы или дает совершенно новые ответы. Итак, теперь у многих студентов есть два набора разных ответов.

Все математические вопросы требуют работы, даже если в вопросе об этом прямо не говорится. Читатели не могут начислять баллы за правильные ответы, если нет хотя бы определенной настройки. В этом году я прочитал вопрос из двух пунктов, в котором студентам требовалось преобразовать молярность и объем в массу. Студенты получали один балл за обнаружение родинок и второй за обнаружение образования.Учащиеся могут легко найти правильные ответы на своем калькуляторе, но если они записали только один из этих результатов (например, 7,91 г), они не получили баллов. Вместо этого они должны были продемонстрировать результаты, такие как 0,10000 л * 0,500M = 0,0500 моль и 0,0500 моль * 158,1 г / моль = 7,91 г (значимые цифры по этому вопросу не оценивались). Студенты получили один балл за начальный расчет молей, а второй за расчет массы.

Если ученикам есть место для рисования диаграмм частиц, структур Льюиса, графиков или любых других изображений, и ученик совершает ошибку в этой области, они могут вычеркнуть это и перерисовать на дополнительном листе.Вычеркнутые ответы никогда не читаются.

Понимание того, что задается в вопросе

Учащиеся могут не понимать подсказки с вопросами. Студентам важно знать, что означают «ключевые подсказки», такие как те, что указаны в таблице ниже. Многие учителя могут обнаружить, что обучение быстрой лексике не входит в сферу применения AP Chemistry, полагая, что ученики уже должны знать, что означают эти термины. Но если вы хотите, чтобы ваши ученики показывали наилучшие результаты, важно обрисовать в общих чертах ожидания каждого запроса, чтобы ученики точно знали, что им нужно дать в своем ответе.В следующей таблице представлен небольшой список подсказок, которые были замечены в последних экзаменационных вопросах. В столбце «Значение» указано, как я объясняю эту конкретную подсказку в моих классах.

Подсказка Значение
Обосновать ответ Покажите всю математическую работу и объясните, как узнать ответ, используя новую информацию, не представленную в вопросе.
Государственное свидетельство Используйте информацию из вопроса (диаграмма данных, график и т. Д.) для подтверждения утверждения и аргументации, включая новые связи, не указанные в вопросе.

ПРИМЕЧАНИЕ: Помните, что «указать доказательства» отличается от «обосновать» в большинстве случаев, потому что вопрос конкретно просит учащихся ссылаться на данную информацию.

Рассчитать Покажите всю работу для каждого ответа, используя соответствующее уравнение, значащие цифры и единицы измерения.
Объяснять Дайте понять, описав более подробно с новыми актуальными фактами.
Представлять Нарисуйте или смоделируйте предоставленное описание.

ПРИМЕЧАНИЕ. Эта подсказка часто сопровождается диаграммой частиц и конкретными указаниями, указывающими, как учащийся должен рисовать частицы, а также сколько частиц нужно рисовать. Студенты должны соблюдать правила, чтобы получить зачет.

Идентифицировать Выберите правильный ответ.

ПРИМЕЧАНИЕ: Вообще говоря, достаточно короткого ответа. Никаких оправданий не требуется.

Оценивать У значения есть диапазон ответов, но он должен находиться в допустимом диапазоне и включать правильные значащие цифры.

ПРИМЕЧАНИЕ: Если стеклянная посуда, показанная в вопросе, показывает значение до сотых долей, оценка также должна иметь значение сотых долей.

С точки зрения Используйте следующие слова в объяснении вместе с новыми релевантными деталями, которые связаны с условиями.

ПРИМЕЧАНИЕ. Попросите учащихся подчеркнуть необходимые термины и, после ответа на вопрос, убедиться, что они использовали эти термины или синонимы.

По графику Найдите на графике доказательства, объясняющие это явление, и объясните свои рассуждения.

Например, я оценил вопрос 1 (e) (ii) экзамена 2018 года, который требовал от студентов преобразовать предыдущий ответ в кДж / моль rxn и «включить соответствующий алгебраический знак в свой ответ». Вызывает тревогу количество студентов, которые получили правильное значение, но никогда не указали правильный отрицательный знак.А поскольку вопрос требовал алгебраического знака, читатели не могли принять термин «выпущен» или присудить полные баллы любому ответу, который также не включал отрицательный знак. Кроме того, в вопросе 1 (d) экзамена студентов спросили: «Каково изменение температуры реакционной смеси, согласно графику?» К сожалению, сотни студентов ответили на этот вопрос, указав что-то вроде «кривая графика увеличивается, а затем выходит на плато», и не получили баллов, потому что они никогда не давали числовых ответов.

Экзамен AP — это правильно. Это то, с чем борются некоторые студенты. Вопрос 1 (f) экзамена: «Величина изменения энтальпии, рассчитанная по результатам второго эксперимента, такая же, как результат, рассчитанный в части (e) (i). Объясните этот результат ». Многие студенты опровергли это утверждение, заявив, что вымышленный студент, описанный в вопросе, вероятно, неправильно выполнил вторую попытку, и в результате не было начислено никаких баллов. Студенты должны были объяснить сравнение, упомянув, что как величина тепла, так и количество молей увеличиваются в один и тот же фактор — и что при разделении для получения кДж / моль факторы отменяются.

Однако, если в другом вопросе учащихся просят «согласиться или не согласиться» с утверждением, именно тогда учащийся может заявить в своем ответе, что утверждение неверно. Например, вопрос 2 (c) экзамена 2018 года гласил: «Студент предполагает, что повышение температуры увеличит количество N 2 O 3 (г) в равновесной смеси. Укажите, согласны вы или не согласны с гипотезой. Обосновать ответ.» Я слышал от других читателей вопроса 2 в этом году, что многие студенты ответили на вопрос расплывчато и никогда не указали, согласны они или нет — таким образом, не зарабатывая баллов.Поэтому убедитесь, что у ваших учеников есть практика с такими вопросами.

Актуальные темы, признанные за последние годы

В целом, когда я размышляю о последних годах переработанного экзамена и разговариваю с другими читателями, кажется, есть несколько подтем, которые авторы заданий AP любят включать. Назвать несколько:

  • pKa = pH в точке половинного эквивалента
  • сравнение неочевидных межмолекулярных сил (когда лондонские дисперсионные силы сильнее дипольных)
  • скелетные электронно-точечные диаграммы Льюиса, требующие добавления электронов
  • Описание и сравнение закона Кулона
  • период полураспада для 1 st реакций порядка
  • фотоэлектрические спектры
  • стеклянная посуда, необходимая для различных лабораторий (обычно кислотно-щелочных лабораторий)
  • Диаграммы частиц
  • графический анализ
  • Анализ лабораторных ошибок

И хотя на экзамене больше нет вопросов, посвященных чистым ионным вопросам, они по-прежнему очень актуальны, дважды появившись в разделе бесплатных ответов в 2018 году.Это не исчерпывающий список; не стесняйтесь добавлять в этот список и обсуждение на платформе AACT.

Я надеюсь, что вы нашли размышления и указатели полезными, и что вы можете включить некоторые идеи в свое собственное обучение. Лично я прочитал экзамен два года подряд и у меня появилось так много новых идей и идей, что я хотел поделиться ими с другими учителями химии. Рекомендую стать читателем (подробности см. посетите веб-сайт College Board)! Большинство учителей, которых я встречал на чтении, говорят, что чтение экзамена по химии — лучший опыт профессионального развития, который у них когда-либо был.

Если у вас есть вопросы или советы по подготовке учеников к экзамену по химии Advanced Placement, поделитесь ими с нами на форуме обсуждения веб-сайта AACT.

Фотография предоставлена:
(обложка статьи) smolaw / Bigstock.com

vcaa экзамены по физике Все. Все студенты, которым необходимо сдать экзамены VCAA, обязаны прочитать и понять правила экзаменов GAT и VCA. Смотрите полный список репетиторов. . 80 долларов. Надеюсь, вы уже знаете о прошлых экзаменационных работах VCAA для модулей 3 и 4 по математическим методам: математические методы (2002–2009) — экзамены и оценка экзаменов 10 ноября 2013 г. · Подкаст VCE Physics on Light & Matter; Подкаст VCE Physics on Exam time; Викторина VCE Physics on Force & Motion; Кирен М. о викторине «Силы и движение»; Недавние Посты.Microsoft Word — 2007 VCAA Physics Exam 2 Solutions. vcaa. Решения VCAA Sample Physics Exam v2 2017 — PDF 141KB. Это заняло все выходные. Решения для пробных экзаменов по физике 2017 — PDF 109 KB. Книга вопросов GAT 2018. Если вы ранее подавали заявку с использованием SSMS, вернитесь к эссе об этосе воина армии. com Решения для экзаменов по физике VCAA, 2014 © Copyright 2014 itute. ГОД 11 ХИМИЯ • В октябрьский тестовый период работа будет состоять из 25 заданий с несколькими вариантами ответов и набора вопросов с короткими ответами.Он содержит все, что вам нужно знать о VCE Physics, и ничего больше. Как мы все знаем, на многих конкурсных экзаменах, таких как SSC, Railways, UPSC и другие государственные PCS, вопросы по физике задаются неоднократно, поэтому вы не можете игнорировать раздел физики общих наук. Новости и данные. экзаменационная работа предоставлена, или если в вашей экзаменационной работе отсутствуют страницы. Эти экзамены отличаются высочайшим качеством и по сложности напоминают экзамен в стиле VCAA. Все экзамены соответствуют учебной программе VCAA на 2016-2021 годы.В комплект поставки входят: Примеры решений для экзамена по физике v2. 20 долларов. VCE Chemistry Unit 3 (12 год) — Пробный экзамен с подробными ответами. doc Автор: joe Дата создания: 01.12.2012 8:46:43 Практические экзамены по физике. VC. Ответы. Добавить в корзину. Программа подготовки к праздничным экзаменам поможет вам тщательно изучить все ключевые области обучения, указанные в Плане исследования VCAA для каждого предмета перед срочным экзаменом. Просмотр графиков 2019 Просмотр графиков 2020 Искусство Доступные в настоящее время должности. Экзаменационные работы написаны так, чтобы имитировать стиль и содержание официальных документов VCAA, и содержат готовые отработанные решения. Оригинальный практический экзамен по физике модулей 3 и 4, написанный для TSFX старшими, опытными экзаменаторами VCAA.Срок аккредитации для блоков 3 и 4 был продлен и также истекает 31 декабря 2022 года. По причине этого, чтобы стоять в недавно оставленном. Вопросы VCAA. Этот веб-сайт содержит краткое изложение материала, соответствующие видео и рабочие листы с решениями, включая вопросы экзамена VCAA. Каждую минуту бодрствования проводите на экзаменах по физике, а каждую минуту сна — на сновидениях об учебниках физики Выполняйте повторные экзамены по разделам 3 и 4 каждую неделю Пробуйте практические экзамены каждую неделю или около того, но ничего больше. Только читайте учебник и задавайте вопросы из учебника. Лекция по физике c 2016 Алвин Ван 5/63 Генератор случайных вопросов.** к *. Предыдущие экзаменационные работы примерно с 2000 года, включая буклеты с ответами, можно найти на веб-сайте Управления по учебным программам и оценке штата Виктория. 17 июля 2017 г. · VCAA не выпускает никаких «официальных» ответов на образцы экзаменов, однако по тем предметам, которые я изучал, многие вопросы взяты из прошлых экзаменов, так что вы можете найти материал таким образом. Прошедшие экзамены VCE и связанный с ними контент можно найти прямо на сайте www. Информация о VCE-VCAA. 16 нояб.2019 г. · Решения для экзаменов по физике VCAA 2019. Обратный заказ: Да.Решения для пробного экзамена 1 по математическим методам 2020 — PDF 68 KB. Важные вопросы по физике и ответы в формате PDF для SSC. Лиго занимается vcaa и современной физикой. 2017-2022 VCAA VCE Physics Sample Exam — Подробные ответы. Использование этих инструментов и УВЕДОМЛЕНИЕ: Задержка проекта исследования физики VCAA Период аккредитации для модулей 1 и 2 был продлен и истекает 31 декабря 2022 года. Продукт включает: 1 экзамен X — документ 1 и полностью готовый экзамен по физике 2018 (NHT) 12 НЕЗАПИСЫВАЙТЕ В ЭТОЙ ЗАПИСИ ONOTWRITEINTHISAREA РАЗДЕЛ B — Вопрос 4 — продолжение Вопрос 4 (9 баллов) Учащиеся перемещают квадратную проволочную петлю из 100 витков с площадью поперечного сечения 4.2016 Бесплатный экзамен по физике VCAA itute. Документы для пробных экзаменов TSSM доступны для 3-го и 4-го блоков физики. отсутствуют страницы. Microsoft Word — Решения для экзаменов по физике VCAA 2018. 50 м –1 с постоянной скоростью 1. au VCE Physics Unit 3/4 (12 год) вместе — «конец года» 2. Прямой упор делается на подробный обзор содержания и возможные вопросы экзамена. Отличный инструмент для пересмотра экзаменов VCE. Генератор случайных вопросов. Моя первая версия VCEasy Visual Physics Units 1 и 2 теперь готова для использования в классе.Информация о французском экзамене. Если вы обнаружите какие-либо ошибки, сообщите нам об этом. Объявление будет сделано до истечения запланированных последних пяти минут написания. Для учителей это план доски и викторины. 2017. Время экзаменов 2021 г .; Понимание света и почему он существует. com Скачать бесплатные экзамены по физике с полными решениями. Экзамены по физике с полными решениями. Частицы и волны: центральная загадка квантовой механики — Чад Орзель; Объясняет профессор Дэйв: Неопределенность Гейзенберга ФИЗИКА Письменный экзамен День Дата Время чтения: *.Heinemann Physics 1 и 2, 4-е издание являются текущими и доступны до тех пор, пока не будет реализован новый дизайн исследования. На этом веб-сайте представлены материалы для VCE Physics Units 1-4 для текущего дизайна исследования. Свидетельство о высшей школе (HSC) Создать множественный. Каждую минуту бодрствования проводите экзамены по физике, а каждую минуту сна мечтайте об учебниках физики. Выполняйте повторный экзамен по разделам 3 и 4 каждую неделю. Выполняйте практические экзамены каждую неделю или около того, но ничего другого. Только читайте учебник. и попробуйте вопросы из учебника. Просто посетите пересмотренную лекцию VCE Physics c 2016 Alwin Wang 5/63 Эпические банки вопросов, чтобы подготовить студентов к экзамену.Вы не могли. Страница 2/4 24 октября 2017 г. · Опубликовано 24 октября 2017 г. автором VCE Physics. 2017 Kilbaha VCE Physics Units 3 and 4 Trial Studyclix. ** (15 минут) Время написания: *. Год. 6 июля 2021 г. · Вот отчет оценщика с экзамена VCAA Physics 3/4 2014 года. Рекомендации к экзаменам на 2021 год: теперь доступны письменные экзамены в ноябре / декабре, чтобы помочь студентам, которые готовятся к сдаче письменных экзаменов VCE в конце года. vcaa. ) Тест 2 с решениями: 3 экзамена | Физика II: электричество и магнетизм с решениями пробных экзаменов по физике 2018 г. — PDF 147 КБ.doc Автор: joe Дата создания: 17.04.2017 17:06:48 2017 VCAA Sample Physics Exam v2 Solutions Некоторые ресурсы VCE Accounting, включая видео, решения, заметки и практические материалы. Systems Eng Информация. pdf. ВНИМАНИЕ: Задержка в разработке исследования по физике VCAA Период аккредитации для блоков 1 и 2 был продлен и истекает 31 декабря 2022 года. 0 × 10–4 м2. Распечатайте эти вопросы. 24 октября 2017 г. · Опубликовано 24 октября 2017 г. автором VCE Physics. nsw. Сложность может варьироваться в разные годы в зависимости от того, что группа решит написать, но они, конечно, не ставят перед собой задачу разработать более легкую или более сложную работу для какого-либо конкретного цикла экзамена.Общий тест на успеваемость (GAT) 21 мая 2020 г. · Учащиеся 12-го класса Виктории начнут сдавать свои выпускные экзамены в начале ноября и получат свои результаты к концу года, поскольку правительство вводит строгие правила гигиены для государственных школ официальный 2020 Экзаменационные работы по физике с полностью проработанными решениями и советами по выставлению оценок. Полный обзор курса и структуры оценивания с советами по изучению по конкретным предметам и советами по экзаменам, дополнительными вопросами с автоматической пометкой с несколькими вариантами ответов и другими цифровыми ресурсами (доступны онлайн через или заказать сеанс).8 октября 2021 г. · Экзаменационные работы — с 2000 г. Экзамены VCAA — смотрите ЗДЕСЬ. Наименьшее расстояние по дуге большого круга между Лимой и Вашингтоном, округ Колумбия, составляет 5697 км. Решения для экзаменов по физике VCAA 2019 — PDF 107 KB. Предыдущие статьи HSC. Эта запись была размещена в Экзамены, Блок 3, Блок 4 и помечена как Экзамены, VCAA. Просмотреть графики 2019 г. Просмотреть графики 2020 г. доступны онлайн через o заказать сеанс).au. Частицы и волны: центральная загадка квантовой механики — Чад Орзель; Профессор Дэйв объясняет: Экзамены по физике VCAA Heisenberg Uncertainty Free VCAA Physics Examination 1 Solutions — PDF 373KB ПРИМЕЧАНИЕ: Спасибо Yaodong Tang, который указал, что увеличение емкости только компенсирует и может сделать СРЕДНЕЕ напряжение на выходе 12 В. Оба поля находятся в одном направлении B. Free vcaa. экзамены vcaa английский / дополнительные экзамены vcaa / экзамены vcaa 2021 / экзамены vcaa по физике / экзамены vcaa по биологии / методы экзаменов vcaa / состояние теста il cdl / holt mcdougal ключ ответа по аналитической геометрии / ответы на экзамен по защите пищевых продуктов / итоговый экзамен по экологической науке ответы / sc мотоциклетное разрешение тестовое руководство / тест по анатомии и физиологии 1-4 пересмотр экзамена Новости физики Принципы практической физики примечания.Физика с Synno. Вы не можете подать заявку через этот экран. Физика Сдавайте практический экзамен 12-го класса на веб-сайте VCAA по математике (дополнительно, методы или специалист), химии, физике, биологии, психологии, здоровью и человеческому развитию, юриспруденции, управлению бизнесом, физическому воспитанию или бухгалтерскому учету. 15:00 (15 минут) Время написания: 9. 7 мс в конце 11 февраля 2015 г. · Подкаст VCE Physics on Light &atter; Подкаст VCE Physics on Exam time; Викторина VCE Physics on Force & Motion; Кирен М. о викторине «Силы и движение»; Недавние Посты.отчет о стрессе бумага электротехнология n3 вопросники меморандум vcaa экзаменационные документы по биологии флорида математика подключает курс 2 глава практика для экзаменационной работы была предоставлена ​​или если ваша экзаменационная работа. . Крейг Тилли был экспертом по физике более 16 лет и является автором A + Physics Oct 9, 2020 · Правила экзаменов VCAA, которые должны знать все студенты. 2013-2016-vcaa-Physics-sample-exam-solutions — Прочтите онлайн бесплатно. Спецификации публикуются для каждого экзамена и содержат полезную информацию об общих условиях ФИЗИКА Письменный экзамен Среда, 14 ноября 2018 г. Время чтения: 9.Экзамен VCAA по физике 2019 Q19 b) ii Меня до смерти тошнит от ужасных объяснений в отчете экзаменатора по физике. com Не делать фотокопии Бесплатно скачать и распечатать с www. Информация о глобальной политике и политике. Работа TSSM по физике модуля 1 написана в формате, аналогичном официальным работам по модулям 3 и 4 VCAA, и содержит готовые отработанные решения, позволяющие студентам при желании самостоятельно исправлять ошибки. Экзамен Ревизия Новости физики Заметки о принципах практической физики. Обратите внимание: содержимое этого веб-сайта предназначено для использования вместе с учебниками.Ниже показан расширяемый список вакансий, на которые в настоящее время принимаются заявки. Веб-сайт VCAA содержит единственные официальные, актуальные и авторизованные версии экзаменационных работ, дизайнов исследований и других оценочных материалов VCAA. Годовой отчет Годовой отчет, контрольный список соответствия и графики консультаций и зарубежных поездок. Решения для экзаменов по физике VCAA 2017 — PDF 96 КБ. Эссе армейского воина. doc Автор: Джо Дата создания: 09.11.2006 13:12:32 Название: Microsoft Word — 2012 VCAA Physics Exam 2 Solutions.5 кОм Название: Microsoft Word — 2009 VCAA Physics Exam 1 Solutions. Трасса автомобиля составляет часть окружности радиусом 44 м, колея горизонтальная. отчет о стрессе бумага электротехнология n3 вопросы вопросы памятка vcaa экзаменационные работы по биологии флорида математика объединяет курс 2 глава практика для читателей предлагается читать больше и учиться дальше. Щелкните по предметной группе, которую хотите просмотреть. Эти два поля находятся в противоположных направлениях. C. Scribd — крупнейший в мире сайт для чтения и публикации в социальных сетях.com 3 Область исследования −−− Взаимодействие света и вещества Q19a E = hf = (с 6. 15 до 11. 2016 Решения: Часть 1 и Текущая программа VCE Physics была обновлена ​​в 2016 (Блоки 1 и 2) и 2017 ( Блоки 3 и 4) У нас есть прошлые документы VCE Physics за период с 2002 по 2019 год, чтобы помочь вам в этом.37 вопросов. Отчет об экзамене VCE Physics за 2017 год (с поправками от 30 октября 2019 года) Экзамены, относящиеся к предыдущим проектам исследований. с введением нового дизайна исследования следующие прошлые экзамены не обязательно являются руководством к текущему экзамену VCE для этого исследования.Информация об экзамене по русскому языку. Документы пробного экзамена являются важным компонентом любой программы пересмотра, особенно в последние недели перед экзаменом. Принципы практической физики — терминология VCAA. Оригинальный практический экзамен по физике модулей 3 и 4, написанный для TSFX старшими, опытными экзаменаторами VCAA. VCAA прошлые документы и ответы GAT. Затем ученики исследуют взаимосвязь между различными исходными переменными и диапазоном R (на ровной поверхности), как показано на Рисунке 9. 00.2017 Бесплатные экзамены по физике VCAA. ** (2 часа 30 минут) ВОПРОСЫ И ОТВЕТНАЯ КНИГА Структура книги Раздел Количество вопросов Количество вопросов, на которые нужно ответить Количество оценок A 20 20 20 B 18 18 110 Всего 130 Прочтите бесплатные экзамены по физике с решениями HVPS), а не 150V / 150В. экзамен по физике vcaa

размышлений читателя экзамена AP по химии

Мне посчастливилось быть отобранным для чтения второго экзамена по химии в Солт-Лейк-Сити, штат Юта, в июне этого года.Позвольте мне поделиться некоторыми советами, которые я узнал.

Чтение экзамена по химии Advanced Placement было чрезвычайно ценным опытом. Быть знакомым с тем, как студенты из разных стран отвечают на эти сложные вопросы по химии, было, вероятно, лучшим профессиональным развитием, которое я когда-либо получал. Как правило, читатели-химики проходят обучение и получают задание ответить на один вопрос из семи вопросов с бесплатными ответами, которые задаются на экзамене в этом году. Иногда читатели будут переведены и переобучены по другому вопросу позже на неделе, в зависимости от того, сколько времени потребуется на оценку вопроса.К концу недели все читатели видели тысячи ответов только на 1-2 вопроса с бесплатными ответами; от коротких, сладких и по существу логичных ответов до запутанных, лишних и образных неправильных ответов. Прочитав тысячи ответов в ходе двух чтений AP Chemistry, я хотел бы поделиться некоторыми советами, которыми учителя AP Chemistry должны рассмотреть возможность поделиться со своими учениками во время курса химии AP, чтобы лучше подготовить их к успешной сдаче экзамена.

Сначала ознакомьте своих учеников с форматом экзамена AP. Исходя из моего опыта, очевидно, что многие студенты идут на экзамен, не понимая, что от них будут спрашивать. Важно, чтобы они понимали формат экзамена, чтобы учесть правильные ожидания и наиболее эффективно спланировать свое время. Я недавно писал о формате экзамена, но вот несколько советов:

  • Раздел I экзамена по химии AP состоит из 60 вопросов с несколькими вариантами ответов за 90 минут.
  • Вопросы переплетены в книгу со справочной таблицей.
  • Раздел II экзамена по химии AP состоит из трех длинных вопросов, каждый из которых занимает в среднем 23 минуты и дает учащимся до десяти баллов каждый, а также четырех коротких вопросов, требующих в среднем девяти минут и дающих до четырех баллов каждый.
  • Каждый вопрос с бесплатным ответом написан на одной-двух страницах, на которых нет места для работы. После всего вопроса в переплетенной книге дается несколько страниц для ответов.

Студентам важно увидеть это форматирование перед экзаменом, чтобы они знали, как они хотят организовать свою работу. Я рекомендую дать студентам практический экзамен, чтобы они имели опыт работы с этой установкой. Вы можете сдать старый экзамен; но имейте в виду, что у студентов есть доступ к ответам в Интернете. Если вы прошли аудит AP, у вас есть доступ к нескольким практическим экзаменам. Эти практические экзамены безопасны; это означает, что они не должны размещаться на каких-либо веб-сайтах или доступны для учащихся дома. Поэтому учителям химии в программе AP важно следить за тем, чтобы тесты проводились только в классе, чтобы другие учителя могли использовать их в качестве достоверной оценки.Мои студенты сдали все практические экзамены в какой-то момент учебного года, и когда я спросил, какой инструмент является наиболее ценным для проверки, большинство студентов упомянули эти практические экзамены. Студенты смогли попрактиковаться в управлении временем и фактически увидеть формат экзамена до дня экзамена.

Учащимся необходимо научиться правильно организовывать свои свободные ответы. Читатели должны прочитать все страницы работы, посвященной оцениваемому вопросу, и следовать ему.Поэтому, если ученик пишет одни ответы рядом с вопросами, а другие — на дополнительной бумаге, мы оцениваем все это. Но есть серьезные проблемы, которые могут возникнуть, когда студенты не знакомы с форматом экзамена и с тем, что от них ожидается.

Как упоминалось ранее, вопросы с бесплатными ответами написаны так, что ученикам очень мало места для выработки ответов в отведенном под вопросом месте. Экзамен не предназначен для того, чтобы студенты завершили работу над экзаменационным буклетом рядом с вопросом (если нет поля для ответа, например, для диаграмм Льюиса).Они должны показывать свою работу и ответы в отведенных местах после каждого вопроса в буклете для ответов. Я понимаю, что студентам может быть сложно постоянно перелистывать страницы вперед и назад, чтобы увидеть вопрос, а затем написать свои ответы. Но это определенно лучше, чем пытаться писать в крошечных промежутках между вопросами. Слишком часто ученики пишут слишком мелко, и читать ответ практически невозможно. Студенты, которые пишут так мелким карандашом, также имеют проблему писать слишком мягко, что делает ответы слишком легкими для чтения.Если читать невозможно, читатели обычно обращаются к ведущему за таблицей, чтобы тот помог им прочитать ответ ученика. Однако, если ответ неразборчив, баллы не начисляются. Вместо того, чтобы писать крошечными буквами, учащиеся должны уметь писать свои работы и ответы в соответствующих областях экзамена после вопроса. Для длинных письменных ответов места между подвопросами совершенно недостаточно. А за длинные математические вопросы ученики, пытающиеся втиснуть свою работу в это маленькое пространство, потеряют баллы, если читатель не сможет найти ключевые числа и логическую работу, которая помечена и понятна.Если ученик хочет убедиться, что он получит максимальное количество баллов, он должен записать все ответы в одном месте и только один раз! Так много студентов написали свои правильные ответы рядом с вопросом, а затем поняли, что у них есть лишняя бумага, и решили передать свою работу. Проблема в том, что большая часть этих студентов неверно переставили ответы или дали совершенно новые ответы. Итак, теперь у многих студентов есть два набора разных ответов. Обычно читателям предлагается принять только второй ответ.В этом году я организовал все свои основные экзамены так, чтобы они выглядели как экзамен AP, таким образом, мои ученики обучаются тому, как лучше всего организовывать свои ответы в течение всего года.

Все математические вопросы требуют работы , даже если в вопросе об этом прямо не говорится. Читатели не могут присуждать баллы за правильные ответы, если не задана хотя бы настройка. В этом году я прочитал вопрос 1 экзамена 2018 года, включая вопрос из двух пунктов, требующий от студентов преобразовать молярность и объем в массу.Студенты получали один балл за обнаружение родинок и второй за обнаружение образования. Правильный ответ можно легко найти в их калькуляторе, но если ученик набрал только 7,91 г, он получил 0/2 балла. Они должны были показать работу (0,10000L x 0,500M = 0,0500 моль и 0,0500 моль x 158,1 г / моль = 7,91 г ПРИМЕЧАНИЕ: значащие цифры не оценивались по этому вопросу ). Студенты получили один балл за начальный расчет молей и второй балл за расчет массы.

Если есть место, где ученики могут нарисовать диаграммы частиц, структуры Льюиса, графики или любой другой рисунок, и ученик совершает ошибку в этой области, он может вычеркнуть ее и перерисовать на дополнительном листе. Зачеркнутые ответы никогда не читаются.

Студенты могут не понимать подсказки с вопросами. Студентам важно знать, что означают ключевые подсказки, например подсказки, представленные в таблице ниже. Многие учителя могут обнаружить, что обучение этому быстрому словарю выходит за рамки программы AP Chemistry; что студенты уже должны знать, что означают эти термины.Но если вы хотите, чтобы ваши ученики показали себя с максимальной эффективностью, важно обрисовать в общих чертах ожидания каждого запроса, чтобы ученики точно знали, что им нужно дать в своем ответе. В следующей таблице представлен небольшой список подсказок, которые были замечены в последних экзаменационных вопросах. «Значение» рядом с каждой подсказкой — это то, как подсказка объясняется в моих классах.

Подсказка

Значение

Банкноты

Обоснуйте свой ответ

Покажите всю математическую работу и объясните, как узнать, что ответ правильный, используя новую информацию, не представленную в вопросе.

Государственные свидетельства

Используйте информацию из вопроса (диаграмма данных, график и т. Д.), Чтобы подтвердить утверждение и предоставить аргументы, включая новые связи, которые еще не были установлены в вопросе

Обратите внимание, что «изложить доказательства» отличается от «обосновать» в большинстве случаев, потому что вопрос конкретно просит учащихся сослаться на данную информацию.

Рассчитать

Покажите всю работу с использованием соответствующего уравнения, значащих цифр и единиц для ответа.

Объяснить

Проясните, описав более подробно с новыми актуальными фактами.

Представлять

Нарисуйте или смоделируйте предоставленное описание.

Диаграмма частиц и конкретные указания часто сопровождают это о том, как рисовать частицы, а также множество частиц для рисования. Студенты должны соблюдать правила, чтобы получить зачет.

Идентифицировать

Выберите правильный ответ.

Вообще говоря, достаточно короткого ответа. Никаких оправданий не требуется.

Оценка

Значение имеет диапазон ответов, но должно быть в приемлемом диапазоне и с правильными значащими цифрами.

Если использованная стеклянная посуда показывает значение до сотых долей, то оценка должна иметь значение сотых долей.

в пересчете на

Используйте следующие слова в объяснении вместе с новыми релевантными деталями, которые связаны с условиями.

Попросите учащихся подчеркнуть требуемые термины и, ответив на вопрос, убедиться, что они использовали эти термины или синоним этого термина.

По графику

Найдите на графике доказательства, объясняющие явление, и объясните свои рассуждения.

Например, я оценил вопрос 1 (e) (ii) экзамена 2018 года, требующий от студентов преобразовать предыдущий ответ в кДж / моль rxn и «включить соответствующий алгебраический знак в свой ответ». Вызывает тревогу количество студентов, получивших правильное значение, но никогда не представивших правильный отрицательный знак. И поскольку вопрос требовал алгебраического знака, читатели не могли принять термин «выпущен».Таким образом, за любой ответ, не включавший отрицательный знак, не начислялись полные баллы. Кроме того, в вопросе 1d экзамена 2018 студентам был задан вопрос : «Каково изменение температуры реакционной смеси, согласно графику?» К сожалению, сотни студентов ответили на этот вопрос, заявив, что «кривая графика увеличивается, а затем выходит на плато», никогда не давая числового ответа.

Экзамен не ошибочный. Это то, с чем борются некоторые студенты.Вопрос 1 (f) экзамена 2018 задан: «Величина изменения энтальпии, рассчитанная по результатам второго эксперимента, такая же, как результат, рассчитанный в части (e) (i). Объясните этот результат ». Многие студенты опровергли это утверждение, заявив, что студент, должно быть, неправильно выполнил вторую попытку, и не присудили студентам никаких баллов. Студенты должны были объяснить сравнение, упомянув, что и величина тепла увеличилась, и количество молей увеличилось в один и тот же фактор, а при разделении для получения кДж / моль коэффициенты аннулировались.Однако, если в другом вопросе учащимся предлагается «согласен или не согласен» с утверждением, это время, когда это утверждение может быть неверным. Например, в вопросе 2 (c) экзамена 2018 года задан вопрос : «Студент предполагает, что повышение температуры увеличит количество N 2 O 3 (г) в равновесной смеси. Укажите, согласны вы или не согласны с гипотезой. Обосновать ответ.» Я слышал от читателей вопроса 2 в этом году, что многие студенты ответили на вопрос расплывчато и на самом деле никогда не писали, согласны они или нет, таким образом, не зарабатывая баллов. Убедитесь, что у ваших учеников есть практика с такими вопросами.

Я надеюсь, что вам понравились размышления и указатели, и вы сможете включить некоторые из идей. Лично я читал экзамен два года подряд и у меня появилось так много новых идей и идей, которыми я пытался поделиться. Рекомендую стать читателем (подробности на CollegeBoard.org)! Большинство учителей, с которыми я встречался во время чтения, говорят, что чтение экзамена по химии — лучшее профессиональное развитие, которое у них когда-либо было.Кроме того, мы с другими читателями AP Chemistry проведем пробное чтение на конференции ChemEd в июле этого года. Если вы заинтересованы в участии, перейдите по ссылке www.chemed2019.com, чтобы узнать подробности.

Высокое кровяное давление у детей и подростков

1. Хансен М.Л., Ганн П.В., Kaelber DC. Недостаточная диагностика артериальной гипертензии у детей и подростков. ДЖАМА . 2007; 298 (8): 874–879 ….

2. Макнис К.Л., Поффенбаргер Т.С., Тернер Дж. Л., Франко К.Д., Сороф Дж. М., Портман Р.Дж.Распространенность гипертонии и предгипертонии среди подростков. Дж. Педиатр . 2007. 150 (6): 640–644.

3. Чен Х, Ван Ю. Отслеживание артериального давления с детства до взрослой жизни: систематический обзор и мета-регрессионный анализ. Тираж . 2008. 117 (25): 3171–3180.

4. Мартино Ф, Пудду ЧП, Pannarale G, и другие. Гипертоническая болезнь у детей и подростков, посещающих липидную клинику. Eur J Pediatr . 2013. 172 (12): 1573–1579.

5. Brady TM, Фивуш Б, Флинн Дж. Т., Парех Р. Способность артериального давления прогнозировать гипертрофию левого желудочка у детей с первичной гипертензией. Дж. Педиатр . 2008. 152 (1): 73–78.

6. Урбина Е.М., Хури PR, Маккой C, Дэниэлс С.Р., Кимбалл Т.Р., Долан Л.М. Сердечные и сосудистые последствия прегипертонии в молодости. J Clin Hypertens (Гринвич) . 2011. 13 (5): 332–342.

7. Райли М., Добсон М, Сен А, Грин Л. Распознавание повышенного АД у детей и подростков: как у нас дела? J Fam Pract . 2013. 62 (6): 294–299.

8. Флинн Дж. Т., Кельбер, округ Колумбия, Бейкер-Смит К.М., и другие. Руководство по клинической практике по скринингу и лечению повышенного артериального давления у детей и подростков [опубликованная поправка опубликована в публикации «Педиатрия».2017; 140 (6): e20173035]. Педиатрия . 2017; 140 (3): e20171904.

9. Хаук Л. Скрининг и лечение высокого АД у детей и подростков: обновленное руководство AAP [Практическое руководство]. Ам Фам Врач . 2018; 97 (8): 543–544.

10. Хури М., Хури П.Р., Долан Л.М., Кимбалл Т.Р., Урбина Е.М. Клинические последствия пересмотренных рекомендаций AAP по детской гипертензии. Опубликован досрочно 5 июля 2018 г. Педиатрия. http: // педиатрия.aappublications.org/content/early/2018/07/03/peds.2018-0245. По состоянию на 25 июля 2018 г.

11. Lurbe E, Agabiti-Rosei E, Cruickshank JK, и другие. Руководство Европейского общества гипертонии по лечению высокого кровяного давления у детей и подростков, 2016 г. Дж Гипертенз . 2016; 34 (10): 1887–1920.

12. Vogt BA. Гипертония у детей и подростков: определение, патофизиология, факторы риска и отдаленные последствия. Current Therap Res . 2001. 62 (4): 283–297.

13. Капур Г, Ахмед М, Пан С, Мицнефес М, Чианг М, Mattoo TK. Вторичная гипертензия у детей с избыточным весом и гипертонической болезнью 1 стадии: отчет Консорциума педиатрической нефрологии Среднего Запада. J Clin Hypertens (Гринвич) . 2010. 12 (1): 34–39.

14. Фолкнер Б., Гиддинг СС, Рамирес-Гарника Г, Wiltrout SA, Запад D, Rappaport EB.Взаимосвязь индекса массы тела и артериального давления у педиатрических пациентов первичного звена. Дж. Педиатр . 2006. 148 (2): 195–200.

15. Арчболд К.Х., Васкес М.М., Гудвин JL, Quan SF. Влияние режима сна и ожирения на повышение артериального давления в течение 5-летнего периода: отчет из исследования Tucson Children’s Assessment of Sleep Apnea Study. Дж. Педиатр . 2012. 161 (1): 26–30.

16. Флинн Дж. Т., Мицнефес М, Пирс С, и другие.Артериальное давление у детей с хронической болезнью почек: отчет исследования «Хроническая болезнь почек у детей». Гипертония . 2008. 52 (4): 631–637.

17. Brady TM, Фивуш Б, Парех Р.С., Флинн Дж. Т. Расовые различия среди детей с первичной гипертонией. Педиатрия . 2010. 126 (5): 931–937.

18. Лоулор Д.А., Наджман Дж. М., Стерн Дж, Уильямс GM, Эбрагим С, Дэйви Смит Г.Связь характеристик родителей, рождения и раннего возраста с систолическим артериальным давлением в возрасте 5 лет: результаты исследования беременности и ее исходов, проведенного Университетом Матер. Тираж . 2004. 110 (16): 2417–2423.

19. Мартин Р.М., Несс АР, Ганнелл Д., Эммет П., Дэйви Смит Г. Снижает ли грудное вскармливание в младенчестве артериальное давление в детстве? Продольное исследование родителей и детей Avon (ALSPAC). Тираж .2004. 109 (10): 1259–1266.

20. Мойер В.А. Скрининг первичной артериальной гипертензии у детей и подростков: рекомендация Целевой группы по профилактическим услугам США. Энн Интерн Мед. . 2013. 159 (9): 613–619.

21. Американская академия семейных врачей. Рекомендации клинико-профилактической службы. Гипертоническая болезнь у детей и подростков. https://www.aafp.org/patient-care/clinical-recommendations/all/hypertension.html. По состоянию на 12 февраля 2018 г.

22. Пикеринг Т.Г., Зал JE, Аппель LJ, и другие. Рекомендации по измерению артериального давления у людей и экспериментальных животных: часть 1: измерение артериального давления у людей: заявление для профессионалов Подкомитета профессионального и общественного образования Совета Американской кардиологической ассоциации по исследованиям высокого артериального давления. Тираж . 2005. 111 (5): 697–716.

23. Luma GB, Spiotta RT. Гипертония у детей и подростков. Ам Фам Врач . 2006. 73 (9): 1558–1568.

24. Флинн Дж. Т., Пирс CB, Миллер ER III, и другие.; Группа изучения хронической болезни почек у детей. Надежность измерения и классификации артериального давления в состоянии покоя с помощью осциллометрического прибора у детей с хронической болезнью почек. Дж. Педиатр . 2012; 160 (3): 434–440.e1.

25. Негрони-Баласквид X, Белл CS, Самуэль Дж, Samuels JA.Достаточно ли одного измерения для оценки артериального давления среди подростков? Опыт скрининга артериального давления у более чем 9000 детей со сравнением подгруппы аускультативных измерений с измерениями ртути. J Am Soc Hypertens . 2016; 10 (2): 95–100.

26. Салис П., Ардиссино Г, Занкетти А, и другие. Возрастные различия в офисном (OBP) и амбулаторном мониторинге артериального давления (ABPM) у детей и подростков с гипертонической болезнью: 8C.03. J Hypertens . 2010; 28 (1): e423 – e424.

27. Stein DJ, Скотт К, Аро Абад JM, и другие. Неблагоприятные исходы в раннем детстве и более поздняя гипертония: данные Всемирного исследования психического здоровья. Энн Клин Психиатрия . 2010. 22 (1): 19–28.

28. Визен Дж., Адкинс М, Фортуна S, и другие. Оценка педиатрических пациентов с легкой и умеренной артериальной гипертензией: результаты диагностического тестирования. Педиатрия . 2008; 122 (5): e988 – e993.

29. Фарпур-Ламберт, штат Нью-Джерси, Аггоун Y, Маршан Л.М., Мартин XE, Herrmann FR, Бегетти М. Физическая активность снижает системное артериальное давление и улучшает ранние маркеры атеросклероза у детей с ожирением в предпубертатном возрасте. Джам Колл Кардиол . 2009. 54 (25): 2396–2406.

30. Ян Цюй, Чжан З, Куклина Е.В., и другие. Потребление натрия и артериальное давление среди детей и подростков в США. Педиатрия . 2012. 130 (4): 611–619.

31. Damasceno MM, de Araújo MF, де Фрейтас RW, де Алмейда ПК, Zanetti ML. Связь между артериальным давлением у подростков и потреблением фруктов, овощей и фруктовых соков — предварительное исследование. Дж. Клин Нурс . 2011. 20 (11–12): 1553–1560.

32. Sieverdes JC, Мюллер М, Грегоски MJ, и другие. Влияние хатха-йоги на артериальное давление, α-амилазу слюны и функцию кортизола среди молодежи с нормальным и предгипертензивным давлением. Дж. Альтернативная медицина . 2014; 20 (4): 241–250.

33. Юнь М, Ли С, Вс D, и другие. Табакокурение усиливает связь повышенного артериального давления с ригидностью артерий: исследование сердца Богалуса. Дж Гипертенз . 2015; 33 (2): 266–274.

34. Jerez SJ, Ковьелло А. Употребление алкоголя и артериальное давление среди подростков. Спирт . 1998. 16 (1): 1–5.

35.McCambridge TM, Бенджамин HJ, Бреннер Дж. С., и другие.; Совет по спортивной медицине и фитнесу. Спортивные занятия детей и подростков с системной гипертонией. Педиатрия . 2010. 125 (6): 1287–1294.

36. Li JS, Бейкер-Смит К.М., Смит ПБ, и другие. Расовые различия в реакции артериального давления на ингибиторы ангиотензинпревращающего фермента у детей: метаанализ [опубликованная поправка опубликована в Clin Pharmacol Ther.2008; 84 (5): 636]. Clin Pharmacol Ther . 2008. 84 (3): 315–319.

37. Райли М, Блюм Б. Повышенное артериальное давление у детей и подростков. Ам Фам Врач . 2012. 85 (7): 693–700.

Экзамен по химии | Химия в Иллинойсе

Студентам, имеющим серьезное химическое образование, предлагается сдать квалификационный экзамен по химии, который может дать зачетные баллы по одному или двум общим курсам химии. Квалификационный экзамен предлагается только в начале осеннего и весеннего семестров.Студентам разрешается сдавать квалификационный экзамен только один раз в семестр. Плата за сдачу квалификационных экзаменов не взимается.

Весной 2022 г. Экзамены по химии

Будет две даты экзамена на квалификацию с отдельной регистрацией.

(экзамен по Chem 102 и Chem 104) 13 января, 12:00 — 15:00, 3039 CIF (учебное заведение в кампусе)

(экзамен Chem 102 и Chem 104) 19 января, 17: 00–20: 00, 3039 CIF (учебное заведение в кампусе)

Вы можете сдавать только один экзамен в день.Вы можете сдать экзамен 102 или 104 только один раз в течение этого экзаменационного периода (январь 2022 г.). Вы можете зарегистрироваться на экзамен Chem 102 и Chem 104, если хотите. Вы не можете зарегистрироваться на оба экзамена Chem 102 или Chem 104 одновременно. Вы не можете пересдать ни один экзамен до августа 2022 года.

Регистрация теперь требуется для экзаменов. Регистрация на экзамен 13 января закроется в 12:00 10 января. Регистрация на экзамен 19 января закроется в 12:00 16 января.

Регистрация начнется в 12:00 21.01.21

Пожалуйста, планируйте внимательно.Вы можете регистрироваться только на один экзамен в день.

Не регистрируйтесь на оба экзамена Chem 102 или Chem 104 одновременно. У нас есть ограничения по количеству мест, и, занимая место, которое вы, возможно, не используете, вы помешаете другому студенту сдать экзамен. Если вы сдали экзамен 102 13 января, вы можете зарегистрироваться на экзамен 104 19 января

Регистрации будут проверены, и двойные регистрации могут быть удалены в случае нарушения правил экзамена.


Квалификационные экзамены проходят успешно.На этих экзаменах учащимся не выставляются оценки. Студентам отправляется электронное письмо, в котором указано, прошел или не прошел. Проходные баллы сообщаются в офис колледжа. За провал квалификационного экзамена штрафа нет.


Информация об экзамене по общей химии

Все квалификационные экзамены сдаются очно и в университетском городке. Студенты ДОЛЖНЫ принести свой ноутбук или планшет для сдачи экзамена. Студенты не могут использовать мобильный телефон для сдачи экзамена. Для этих экзаменов нет дистанционной возможности.

Экзамен Chem 102 является экзаменом с несколькими вариантами ответов и предназначен для проверки уровня владения основными темами, охватываемыми учебной программой по общей химии Университета Иллинойса. Он предоставляет 3 часа кредита, эквивалентного Chem 102.

Если студент имеет 3 часа зачетных единиц Chem 102 (зачет AP, зачет квалификации или другой эквивалентный зачет курса), то он имеет право на сдачу экзамена Chem 104 (полный).

За лабораторные курсы общей химии зачетные баллы не предоставляются.

Студентам предоставляется таблица Менделеева, стандартные формулы и константы.

Мы не предоставляем для изучения предыдущие экзамены или другие экзаменационные материалы. Любой учебник химии для первого года обучения обеспечит достаточные ресурсы для обучения.

Время, отведенное для сдачи квалификационных экзаменов

  • Chem 102 Proficiency Exam: 1 час 50 минут
  • Chem 104 Proficiency Exam: 3 часа

Студенты с документально подтвержденным проживанием в офисе DRES кампуса, желающие сдать квалификационный экзамен (ы) по химии, должны предоставить Dr.Келли Марвилл ([email protected]) с документами о проживании до крайнего срока регистрации и настоятельно рекомендуется связаться как с DRES, так и с Киной Финни ([email protected]) по крайней мере за две недели до запланированной даты экзамена, чтобы организовать экзамен. Из-за требований к безопасности экзамена квалификационный экзамен может быть сдан только в одну из запланированных дат данного семестра. В связи с изменением условий доступа к учебным заведениям студенты, которым требуется размещение в DRES, должны зарегистрироваться заранее и незамедлительно связаться с нами.

Результаты экзамена отправляются по индивидуальной электронной почте каждому сдавшему экзамен. Экзамен только сдан / не пройден.

Никаких оценок или оценок не записывается. В офисы колледжа подаются только пропуска. О сбоях не сообщается.

Из-за требований к безопасности экзамена квалификационный экзамен может быть сдан только в одну из запланированных дат данного семестра.

Если после внимательного прочтения приведенной выше информации возникнут дополнительные вопросы, касающиеся экзаменов по общеобразовательной химии, пожалуйста, направьте их по адресу:

Кина Финни, kbaumgar @ ilinois.edu

Food Chemistry — Journal — Elsevier

Food Chemistry имеет три сопутствующих журнала с открытым доступом: Food Chemistry: X , Food Chemistry: Molecular Sciences и Food Chemistry Advances .

Цели и сфера применения Food Chemistry ежегодно оцениваются и модифицируются, чтобы отразить изменения в данной области. Это означает, что темы исследований, которые ранее рассматривались в рамках области охвата, могут теперь выходить за рамки журнала, поскольку наше научное и техническое понимание областей развивается, а темы становятся менее новыми, оригинальными или актуальными для Food Chemistry.

Food Chemistry публикует статьи, касающиеся развития химии и биохимии пищевых продуктов или используемых аналитических методов / подходов. Все статьи должны быть ориентированы на новизну проведенных исследований.

Исследования, продвигающие теорию и практику молекулярных наук о пищевых продуктах или лечения / профилактики болезней человека, не будут рассматриваться для включения в Food Chemistry.

Темы, представленные в Food Chemistry , включают:

— Химия, относящаяся к основным и второстепенным компонентам пищевых продуктов , их питательным, физиологическим, сенсорным, вкусовым и микробиологическим аспектам;

Биоактивные компоненты пищевых продуктов, включая антиоксиданты, фитохимические вещества и ботанические вещества.Данные должны сопровождать достаточное обсуждение, чтобы продемонстрировать их отношение к продуктам питания и / или химическому составу пищевых продуктов;

— Химические и биохимические изменения состава и структуры молекул, вызванные переработкой, распространением и домашними условиями;

Влияние обработки на состав, качество и безопасность пищевых продуктов, других биоматериалов, побочных продуктов и отходов переработки;

–Химия пищевых добавок, , загрязнителей, и других агрохимикатов, а также их метаболизм, токсикология и судьба пищевых продуктов.

Аналитические статьи, касающиеся микробиологических, сенсорных, пищевых, физиологических аспектов, а также аспектов аутентичности и происхождения пищевых продуктов. Статьи должны быть в первую очередь посвящены новым или новым методам (особенно инструментальным или быстрым) при условии, что описана адекватная валидация, включая достаточные данные по реальным образцам, чтобы продемонстрировать надежность. Также будут рассмотрены документы, касающиеся значительных улучшений существующих методов или данные о применении существующих методов к новым продуктам питания или товарам, произведенным в неучтенных географических районах.

Для аналитических статей, особенно тех, которые посвящены разработке и валидации методов, авторам рекомендуется следовать международно признанным руководящим принципам, таким как EURACHEM — для химических соединений (https://www.eurachem.org/index.php/publications/ guides / mv) или FDA — для микробиологических данных (https://www.fda.gov/downloads/ScienceResearch/FieldScience/UCM298730.pdf) и следует применять соответствующие статистические методы. Особое внимание следует уделить линейности, селективности, определению LOD / LOQ, повторяемости и воспроизводимости анализа.Авторам также следует обращать внимание на правильность и, по возможности (количественные методы), определять неопределенность измерения. В целом, реальные образцы должны быть проанализированы новейшим и недавно разработанным методом для целей валидации.

— Будут рассмотрены методы определения как основных, так и второстепенных компонентов пищевых продуктов, особенно питательных веществ и непитательных биоактивных соединений (с предполагаемой пользой для здоровья).

— Результаты исследований по взаимному сравнению методов и разработки стандартных образцов пищевых продуктов для использования в анализе компонентов пищевых продуктов;

— Методы, касающиеся химических форм в продуктах питания, биодоступности питательных веществ и статуса питания;

— Общая аутентификация и происхождение [e.

Добавить комментарий

Ваш адрес email не будет опубликован. Обязательные поля помечены *